Download A 3-year-old girl is brought to the emergency department by her

Document related concepts

Medical ethics wikipedia , lookup

Patient safety wikipedia , lookup

Dysprosody wikipedia , lookup

Adherence (medicine) wikipedia , lookup

Electronic prescribing wikipedia , lookup

List of medical mnemonics wikipedia , lookup

Transcript
A 3-year-old girl is brought to the emergency department by her mother
because of "fever and a rash." When asked to give a more detailed history,
the mother has difficulty providing any additional information. The mother
appears tearful and the child cannot take her eyes off the floor. You are
unable to engage the child in any conversation. Her temperature is 36.7 C
(98.0 F). Physical examination is unremarkable. The most appropriate next
step is to
A. admit the child to the hospital for evaluation and protection
B. ask if there is anyone else in the house that is sick
C. ask the mother and child separately what is concerning them
D. obtain a psychiatry consult immediately
E. send the child home and arrange for a family assessment on a
home visit
Explanation:
The correct answer is C. Because the mother brought her child to the
emergency department because of a nonexistent fever and rash, it seems
like there is something else that is going on. You also need to be able to
interpret a patient's posture and facial expression. The mother is tearful and
the child is looking at the ground; this makes it seem like there are other
issues that need to be explored. There could be many things going on, but
child abuse and domestic violence should be high on your differential. Abuse
and violence often present in vague and indirect ways with complaints of
abdominal pain, headaches, and other symptoms. After talking to both
together, you need to interview each individually because this may allow you
to obtain more information. Direct and nonjudgmental questions are often
helpful in eliciting information.
At this time, you first need to try to obtain more information by talking to
each individually before you assume that this is a case of child abuse or
domestic violence and considering admitting the child to the hospital for
evaluation and protection (choice A).
Since this patient's physical examination is completely normal, it seems
more appropriate at this time to ask the mother and child separately what is
concerning them, as opposed to asking if there is anyone else in the house
that is sick (choice B).
As a physician you should be able to try to elicit what is concerning these
individuals and while a psychiatry consult may be necessary in the future, it
is not immediately (choice D) necessary.
At this time you should try to determine what is concerning these individuals
before you jump to any conclusions about abuse or anything else. Also, if
your suspicions of abuse are high, you should not send the child home with
the parents under any circumstances. Sending the child home and arranging
for a family assessment on a home visit leaves the child open for continued
1
abuse (choice E). The physician should take steps to both protect the child
and investigate further. If you do not believe that there are any signs of
abuse or any other causes for concern, then a family assessment on a home
visit is not necessary. Either way, this is an incorrect answer at this time.
A 26-year-old Spanish-speaking woman presents to your clinic for a bruised left arm.
The patient is a local woman who speaks no English and has recently moved to this
area from overseas. She works at the local telephone company and has moved in with
her fiancee. This is her fourth visit to the clinic for various bruises and injuries. On this
visit she has a diffuse ecchymoses and purpura over her entire left arm and forearm in
the shape of “hand prints.” She claims that she fell while walking down the stairs. On her
previous visits, she has had similar bruises on her chest, abdomen, and back, all of
which she attributed to various “falls”. She is otherwise healthy, denies tobacco or
alcohol. She does report that he fiancee drinks ethanol to intoxication quite frequently
and has recently lost his job. She is somewhat tearful when mentioning him. Her
physical examination is remarkable for the left arm bruising as above, various bruises in
her back and abdomen is various stages of healing, and a valgus deformity of her left
leg. The remainder of her examination is unremarkable. The most appropriate course of
action is to
A. ask the patient in simple, direct language if she has been hit by her fiancee
B. contact the police for intervention
C. no action is indicated as the patient has no active medical issues
D. no action is indicated as the patient will be offended and leave the clinic
E. question the patient in the presence of other health care workers in the clinic
Explanation:
The correct answer A. Ask the patient in simple, direct language if she has been hit by
her fiancee. Because of the high prevalence of domestic violence (DV) and the wide
range of clinical settings in which it can be seen, routine screening must be a part of
preventative care. A number of studies have demonstrated reasonable sensitivity for the
Partner Violence Screen that consists of three simple and direct questions.
Contacting the police (choice B) is not appropriate given that it has not even been
established that this patient suffers from DV and you should first try to address the issue
with the patient.
There are a number of barriers to effective physician screening of DV; the belief that no
action is indicated as the patient has no active medical issues (choice C) as well as that
no action is indicated as the patient will be offended and leave the clinic (choice D) are
among the more common reasons.
It is vital to make the patient feel safe and secure and to question them in private (choice
E).
2
A 32-year-old woman comes to the office "for a prescription of propranolol for stage
fright." She tells you that she is professional singer and lately she has been experiencing
"butterflies" and palpitations before performances. She has been so worried about
having one of these symptoms that she is having trouble sleeping at night. She tells you
that a friend of hers has a similar problem and propranolol has "cured her." She has
been a patient of yours for the past 10 years and you remember that she has severe
asthma, requiring many hospitalizations, the most recent being 2 weeks ago. Her
asthma attacks have been increasingly more severe and have been occurring at an
increased frequency. She tells you that she is in a rush and all she needs is the
prescription. The most appropriate next step is to
A. administer a pulmonary function test
B. explain that propranolol is not a good drug for her
C. give her a referral to a psychiatrist
D. order a chest x-ray
E. prescribe propranolol for her to take before her performances
Explanation:
The correct answer is B. This patient most likely has performance anxiety, which is a
form of social phobia. The treatment usually involves beta-blockers before a
performance to decrease the symptoms. However, a patient with severe asthma should
avoid beta-blockers because they can cause bronchoconstriction and precipitate into an
asthmatic attack.
A pulmonary function test (choice A) and a chest x-ray (choice D) are not indicated at
this time. You already know that she has asthma that has required hospitalizations and
the results of these tests are unlikely to change your management.
A referral to a psychiatrist (choice C) may be helpful in treating her performance anxiety,
but she is in your office for propranolol, so it is your responsibility to first try to explain to
her that her asthma makes her a bad candidate for this treatment.
You should not prescribe propranolol for her to take before her performances (choice E)
because she has severe asthma, which makes beta-blockers a dangerous medication
for her. Beta-blockers can cause airway obstruction, which may lead to worsening
asthma.
A 23-year-old woman is admitted to the hospital from home for altered mental
status. On evaluation in the emergency department, the patient was found to
have severe hypernatremia, a serum sodium of 161 mEq/L. There is no other
past medical history or allergy history available. The patient appeared pale
and profoundly dehydrated. She had evidence of prior scars, possibly
surgical, on her abdomen, chest, and arms. The patient was admitted to the
hospital and over the next few days the patient's hypernatremia was
corrected and her metabolic parameters normalized. She was able to give a
more detailed history about her social situation. She describes feeling very
3
depressed lately and having had an argument with her new boyfriend several
days prior to her admission. You suspect that she may be a victim of
domestic abuse. The most appropriate initial step in addressing your concern
is to
A. ask her for the number of her boyfriend to address your concerns
with him directly
B. ask her to offer more details about the nature of her relationship
with her new boyfriend
C. explain to her that her relationship is obviously not having a positive
impact on her life
D. refer her case to the department of social services in obligation of
your reporting duty as a physician
E. refer her to a psychiatrist who specializes in domestic abuse
Explanation:
The correct answer is B. The concern for potential abuse must be addressed
as any other new piece of clinical suspicion. The best initial step is to try to
gather more information in a non-threatening way that is comfortable for the
patient. While the physician will ultimately want to ask her directly if she is
being abused, it is often better to let that disclosure “fall out” from a
discussion about the particulars of the relationship.
While physicians often feel the desire to “take things in their own hands” to
address alleged abusers directly (choice A), this approach can be extremely
dangerous for the abused party. Abusers obviously dread be discovered,
and are likely to act with retribution against the abused party if they are
confronted by a doctor.
Explaining that her relationship is not having a positive impact on her life
(choice C) is presumptuous without first exploring all of the particulars of the
relationship.
Physicians in most states are obligated to report (choice D) potential abuse
only in the case of children under 18 and adults older than 65. There is no
obligatory reporting for domestic abuse not falling within these parameters.
While the patient may respond very well to treatment from a psychiatrist who
specializes in domestic abuse (choice E), this referral, like any other, should
be made after an earnest attempt to gather the clinical and social details
from the patient by the referring physician.
A 27-year-old African American man is brought to the emergency department
by his girlfriend stating, "I can't swallow." He tells you his symptoms started 2
hours ago with a feeling that his head was "pulled to the side" and his neck
was "tight". Since then he has had difficulty speaking and eating. His medical
history is significant for a remote appendectomy. A chart review reveals that
he has no allergies, was diagnosed with schizophrenia last year, and has
been resistant to taking oral medications. He denies hallucinations and his
4
girlfriend confirms the "voices" are controlled with the injections he receives
each month. His last injection was yesterday. His temperature is 37.0 C (98.6
F), blood pressure is 110/70 mm Hg, pulse is 70/min, and respirations are
26/min. He is an anxious appearing man with excessive drooling. His head
seems fixed in an unusual position and you palpate spasms in several neck
muscles. The most appropriate next step in management is to administer
A. benztropine, intramuscularly
B. benztropine, orally
C. diphenhydramine, orally
D. haloperidol, intramuscularly
E. haloperidol, orally
Explanation:
The correct answer is A. This patient has acute dystonia, which is a brief or
prolonged muscle spasm, usually of the head or neck muscles including the
larynx and pharynx. He most likely received haloperidol or a similar highpotency neuroleptic in his injection yesterday and is experiencing an acute
dystonia as a medication side effect.
Oral benztropine (choice B) and diphenhydramine (choice C) are reasonable
choices for dystonic reactions but this patient's airway is compromised and
he may aspirate his secretions. For this reason, he needs to be given
benztropine intramuscularly and his symptoms will improve in about 30
minutes.
Haloperidol (choices D and E) is not indicated as his schizophrenic
symptoms are controlled. He most likely received haloperidol or a similar
high-potency neuroleptic in his injection yesterday and is experiencing an
acute dystonia as a medication side effect.
A 17-year-old girl is brought to the office by her mother because she has
missed “many periods”. The girl admits to binge eating and exercising in
order to prevent weight gain. She tells you that she is definitely not pregnant,
because she has not had any sexual relations in the past 11 months and
thinks she is not getting her menstrual period because of the excessive
physical exercise she has been doing in the past several weeks. Physical
examination is significant for bradycardia and significant weight loss
compared to the last year. A pregnancy test is negative.At this time you
should order
A. amylase
B. BUN and creatinine
C. liver function tests
5
D. serum potassium
E. thyroid function tests
Explanation:
The correct answer is D. Bulimic patients frequently engage in
compensatory behaviors to prevent weight gain. Those include self-induced
vomiting, abuse of diuretics, laxatives, enemas, or diet pills. The metabolic
disorders frequently seen in these patients are, mostly hypokalemia and
hypomagnesemia.
Amylase (choice A) is not a routine test ordered in bulimic patients. If there
has been evidence of long starvation and suspicion of other medical
conditions, its level might alter.
BUN and creatinine (choice B) can be increased if there are signs of
dehydration, secondary to the abuse of diuretics. Otherwise, these values
should not be changed significantly.
Liver function tests (choice C) are usually not changed in bulimic patients.
They can be ordered to rule out other medical conditions if necessary.
Thyroid function tests (choice E) should be done as a part of regular workup
of patients presenting with this clinical picture. It is not, however, the first to
be ordered.
A 20-year-old college student comes to the clinic because her parents told
her that they are going to make her come home if she does not "get help."
She says that she is not sure what her parents are so concerned about. She
exercises "regularly", gets good grades in school, has a couple of fraternity
guys that she goes out with, and drinks 2-3 beers on the weekends at parties.
She believes that she has a "typical college lifestyle." Her menstrual periods
occur at regular 29-day intervals and have been normal. Her blood pressure
is 120/80 mm Hg, pulse is 65/min, and respirations are 15/min. She is 157 cm
(5 ft 2 in) tall and weighs 54.1 kg (119 lb). Physical examination shows many
dental caries, periodontal disease, pharyngeal abrasions, nail changes, and
multiple, linear lacerations on her forearms in various stages of healing.
Laboratory studies show:
Sodium
139 mEq/L
Potassium 2.8 mEq/L
Chloride
94 mEq/L
Bicarbonate 32 mEq/L
The most appropriate question to help establish a diagnosis is:
A. "Do you ever feel guilty about drinking alcohol?"
B. "Do you typically restrict your diet to under 800 calories a day?"
6
C. "Have you ever taken laxatives as a way to lose weight?"
D. "Have you ever consumed large quantities of food and then
regurgitated it to prevent weight gain?"
E. "Please describe your exercise routine"
Explanation:
The correct answer is D. This patient most likely has bulimia nervosa, which
is an eating disorder characterized by recurrent episodes of binge eating
followed by a compensatory behavior to prevent weight gain (vomiting,
exercise, laxative abuse). Other features include stealing food, alcohol and
drug abuse, self-mutilation, and depression. The individuals are usually at or
slightly over the normal weight for their body, sexual activity is normal or
increased, and they continue to menstruate. Clinical findings that are caused
by recurrent vomiting include dental caries, periodontal disease, pharyngeal
lacerations, and nail changes. Metabolic alkalosis and hypokalemia are
often present and are due to repeated vomiting. Complications include
aspiration and rupture of the esophagus or stomach.
"Do you ever feel guilty about drinking alcohol?" (choice A) is a question that
would be asked if you suspected alcohol abuse. While alcohol abuse has
been associated with bulimia nervosa, it does not seem that this patient is
an alcoholic.
"Do you typically restrict your diet to under 800 calories a day?" (choice B)
would be an indication of anorexia nervosa, which is characterized by the
refusal to maintain a normal, healthy body weight and the disturbance of
body image. These individuals have an intense fear of gaining weight and
therefore restrict food intake to dangerously low levels. Women become
amenorrheic, have decreased sexual desire, ritualized exercise routines,
and changes in their skin and hair. Complications include ventricular
tachyarrhythmias.
"Have you ever taken laxatives as a way to lose weight?" (choice C) and
"Please describe your exercise routine" (choice E) are questions that should
be asked to both anorexics and bulimics. Laxative abuse and exercise
rituals are common in both diseases but laxative abuse is probably more
common in bulimia, while ritualized exercise is more common in anorexia.
This patient has the clinical findings associated with vomiting and therefore
the question about binge eating and purging would more likely establish a
diagnosis.
A 29-year-old attractive married woman has been calling your office several
times in the past week. Her physician is out of town, and you, as his younger
colleague, are covering in his absence. She has called twice at the end of
office hours complaining of vague symptoms. She also appeared in the office
without an appointment, insisting she needed a physical examination,
because she has a new lump on her breast. You provided the examination in
the presence of a female nurse as a chaperone and found nothing. A review
7
of her previous medical records shows no significant medical problems. She
appears again after several days at the office demanding to be seen, just
after all of the office staff has left. She tells you that she has established trust
in you and admires your patience to listen to people's problems. She shares
that she had never experienced that with your colleague and questions if she
could switch providers. The most appropriate intervention at this time is to
A. bring her into the office and call her husband letting him know about
her need for understanding
B. point out that she is only attention-seeking and there is nothing
wrong with her
C. reassure her calmly that she will be seen at the next appointment
given during office hours
D. refuse to talk to her and tell her that you are going to call security to
escort her out
E. see her since she has shown up, even though it is after normal
office hours
Explanation:
The correct answer is C. Any patient is entitled to a thorough medical
evaluation. However, they should be respectful of boundaries. Patients
should be seen when others are present in the office to avoid potential
conflicts and accusations, especially because these patients tend to be
manipulative.
Staying alone with the seductive patient in the office (choice A) and calling
her husband, are not appropriate interventions. They are not aimed at
helping the patient, and may cause more damage to both parties.
Pointing out attention-seeking behavior (choice B) even though it may be
true, is not an appropriate intervention. In order to change it, the patient
needs to understand the origins of such needs.
Refusing to talk to her (choice D) is unacceptable and unethical. Even if this
is only attention-seeking behavior, it indicates the presence of a certain
pathology, and needs to be addressed appropriately. Security should be
called though if the patient still adamantly refuses to leave after a calm and
direct suggestion to schedule an appointment during office hours or go to
the emergency room if there is anything urgent.
After hours consultations (choice E) are not appropriate. This patient
crossed the boundaries several times and most probably will take advantage
of such a situation. In a case of emergency, a referral to the emergency
room should be made and the physicians there should be contacted.
A 64-year-old chronic alcoholic is admitted to the hospital for the evaluation of
an occult gastrointestinal bleed and anemia. He has been a heavy drinker for
years and reports numerous admissions to the hospital because of injuries,
seizures, and other medical problems related to alcohol abuse. He gives a
8
history of "the shakes" when he does not drink and a history of having been
in delirium tremens a few times. On the CAGE questionnaire he scores a 4
out of 4. The patient is also a heavy smoker and has been diagnosed with
COPD. The team is aware that this patient may need to be detoxified
because he might end up developing withdrawal symptoms while in the
hospital. His physical examination is significant for an enlarged liver and
spider angiomata, as well as polyneuropathy secondary to alcohol abuse. His
liver function tests show values 4 times greater than normal. Given this
patient's medical condition, the most appropriate first-line detoxification agent
is
A. carbamazepine
B. chlordiazepoxide
C. disulfiram
D. lorazepam
E. phenobarbital
Explanation:
The correct answer is D. Lorazepam is a short acting benzodiazepine with
no active metabolites. Lorazepam is metabolized to the glucuronide form
and excreted by the kidneys. The drug is tapered over 4-6 days for
detoxification purposes. It is safe in patients with severe liver damage, and it
won't compromise respiration in severe COPD.
Carbamazepine (choice A) is an anticonvulsant that, according to several
studies, is as effective as benzodiazepines in the control of symptoms
associated with alcohol withdrawal. The potential risk of adverse side effects
including the induction of liver enzymes, limits its clinical usefulness for this
application.
Chlordiazepoxide (choice B) is a long-acting benzodiazepine that is used for
detoxification from alcohol in uncomplicated cases. It has several
metabolites that are long acting, thus making it difficult to efficiently manage
the detoxification without risking the accumulation of drug and its
metabolites because of impaired liver metabolism. In patients with severe
COPD, it can compromise respiration secondary to sedative effects.
Disulfiram (choice C) inhibits the enzyme aldehyde dehydrogenase leading
to elevated levels of acetaldehyde. It has been used for long-term treatment
of alcoholism in order to maintain abstinence, but it has not been used for
detoxification.
Phenobarbital (choice E) is like all barbiturates metabolized through liver,
and causes the induction of hepatic enzymes. It can be used for
uncomplicated detoxification from other barbiturates or benzodiazepines, but
has not been used for detoxification from alcohol.
9
A 3-year-old girl is brought to the emergency department by her mother
because of "fever and a rash." When asked to give a more detailed history,
the mother has difficulty providing any additional information. The mother
appears tearful and the child cannot take her eyes off the floor. You are
unable to engage the child in any conversation. Her temperature is 36.7 C
(98.0 F). Physical examination is unremarkable. The most appropriate next
step is to
A. admit the child to the hospital for evaluation and protection
B. ask if there is anyone else in the house that is sick
C. ask the mother and child separately what is concerning them
D. obtain a psychiatry consult immediately
E. send the child home and arrange for a family assessment on a
home visit
Explanation:
The correct answer is C. Because the mother brought her child to the
emergency department because of a nonexistent fever and rash, it seems
like there is something else that is going on. You also need to be able to
interpret a patient's posture and facial expression. The mother is tearful and
the child is looking at the ground; this makes it seem like there are other
issues that need to be explored. There could be many things going on, but
child abuse and domestic violence should be high on your differential. Abuse
and violence often present in vague and indirect ways with complaints of
abdominal pain, headaches, and other symptoms. After talking to both
together, you need to interview each individually because this may allow you
to obtain more information. Direct and nonjudgmental questions are often
helpful in eliciting information.
At this time, you first need to try to obtain more information by talking to
each individually before you assume that this is a case of child abuse or
domestic violence and considering admitting the child to the hospital for
evaluation and protection (choice A).
Since this patient's physical examination is completely normal, it seems
more appropriate at this time to ask the mother and child separately what is
concerning them, as opposed to asking if there is anyone else in the house
that is sick (choice B).
As a physician you should be able to try to elicit what is concerning these
individuals and while a psychiatry consult may be necessary in the future, it
is not immediately (choice D) necessary.
At this time you should try to determine what is concerning these individuals
before you jump to any conclusions about abuse or anything else. Also, if
your suspicions of abuse are high, you should not send the child home with
the parents under any circumstances. Sending the child home and arranging
for a family assessment on a home visit leaves the child open for continued
10
abuse (choice E). The physician should take steps to both protect the child
and investigate further. If you do not believe that there are any signs of
abuse or any other causes for concern, then a family assessment on a home
visit is not necessary. Either way, this is an incorrect answer at this time.
A 29-year-old woman who you have been treating for bipolar disorder comes
to the office because of feelings of "sadness" over the past few months. She
has been taking lithium carbonate for 3 years and has not had any "periods of
sadness" or manic episodes requiring hospitalization since starting therapy.
Before that time, she had been hospitalized 4 times for manic episodes. She
feels "sluggish" and tired and has difficulty concentrating at the office. Her
skin is dry and cool, but the remainder of the examination is unremarkable.
Laboratory studies show:
With regard to her drug regimen, the most appropriate action at this time is to
A. add St. John's wort
B. add fluoxetine to her current therapy
C. begin levothyroxine therapy
D. discontinue lithium carbonate therapy
E. lower the dose of lithium carbonate
F. make no changes in therapy
Explanation:
The correct answer is C. This patient has hypothyroidism, which is a known
side effect of lithium carbonate therapy. It occurs in approximately 5-9% of
patients taking the drug and for this reason, routine testing of thyroid
function is recommended at least every year during therapy. In addition to
routine testing, hypothyroidism should be ruled out in any patient who
experiences depressive symptoms during lithium therapy. Treatment with
levothyroxine is indicated in patients on lithium therapy who develop
symptoms of hypothyroidism. Some believe that treatment for asymptomatic
thyroid dysfunction is also necessary. Since this patient has laboratory
11
studies consistent with hypothyroidism, signs and symptoms including a sad
mood, difficulty concentrating, "sluggishness", and cool, dry skin, she should
be treated.
It is incorrect to add St John's wort (choice A) to her current therapy because
she has symptomatic hypothyroidism that needs to be treated with thyroid
replacement therapy. St. John's wort is an herbal medication that many
people believe is effective in the treatment of depression. It is not FDA
approved.
Adding fluoxetine (choice B) is the incorrect treatment for this patient's
"sadness" because she has hypothyroidism, which is likely either causing or
contributing to this mood disorder. Treating the hypothyroidism is the most
appropriate next step.
Since this patient has symptomatic hypothyroidism, she needs to be treated
with levothyroxine and it is inappropriate to discontinue lithium carbonate
therapy (choice D). This patient needs the lithium for her bipolar disorder,
especially because she required hospitalizations in the past.
Lowering the dose of lithium carbonate (choice E) is not appropriate. This
patient's bipolar disorder is being adequately treated with the dose that she
is taking and her levels are within the therapeutic range. Since thyroid
dysfunction is a known side effect of this therapy it is important to watch for it
and treat it, but lowering the dose in her case is not necessary.
As stated above, this patient has symptomatic hypothyroidism and should be
treated with levothyroxine, therefore making no changes in her therapy
(choice F) is incorrect.
A 20-year-old comes to the clinic because of problem that has bothered her
for "a while". She has had difficulty throughout her academic career in spite of
studying an average of 5 hours per night and taking many extra sessions of
tutoring in her classes. She is not able to concentrate due to having to
continually check that the door in her apartment or study area of the library is
locked. She checks to see that it is locked an average of 15 times an hour
while trying to study. She is fully aware each time she checks that the door is
locked behind her, but cannot resist the temptation to check to make sure.
She sometimes counts to 100 backward and forward after checking that her
door is locked in order to distract herself, however, this only provides
temporary relief. There is no history of abuse, no history of hallucinations,
and the patient states overtly that she does not fear for her safety while
checking her door. An appropriate medication to treat this patient's condition
is
A. haloperidol
B. lithium
C. lorazepam
12
D. paroxetine
E. valproic acid
Explanation:
The correct answer is D. This patient suffers from obsessive-compulsive
disorder and demonstrates compulsive checking rituals that she
acknowledges are not grounded in reality-based concerns. The treatment of
choice for obsessive-compulsive disorder is higher dose selective serotonin
reuptake inhibitors such as paroxetine, fluvoxamine, sertraline, citalopram,
and fluoxetine.
Haloperidol (choice A) is an antipsychotic medication that has no indication
in the treatment of obsessive-compulsive disorder.
Lithium (choice B) is used in the treatment of bipolar disorder and may be
used as an augmentation strategy for the treatment of unipolar depression. It
has no indication, however, for the treatment of obsessive-compulsive
disorder.
Lorazepam (choice C) is an anxiolytic and antiepileptic medication that is
effective for treatment of some anxiety disorders such as panic disorder and
generalized anxiety disorder. However, it is not indicated in the treatment of
obsessive-compulsive disorder.
Valproic acid (choice E) is an anticonvulsant used in the treatment of bipolar
disorder. It has no indication in the treatment of obsessive-compulsive
disorder.
A 27-year-old woman who was trained as a psychiatrist comes to the office
for the first time because of recurrent intermittent abdominal pain. She
complains of "vague" pain that is usually present at night and typically
resolves by mid-morning the following day. She denies any change in bowel
habits, blood per rectum, or weight loss. Her husband is a professional
football player and he was recently transferred to your city's team from across
the country. The patient is tearing small pieces from a tissue that she is
holding as she relays her past medical history, which is unremarkable. She
does not have any children yet, but tells you that her husband wants her to
have a "son" for him soon. Abdominal examination is unremarkable. There is
a 2 cm yellowish ecchymosis on her left buttock and a 3.2 cm purple
ecchymosis on her right upper arm. When asked about these findings, she
tells you with a clenched fist and watery eyes averted to the ground that she
is "accident-prone." The most appropriate remark at this time is:
A. "For such a smart woman, you seem to have gotten yourself in a
dangerous position."
B. "I can't help but notice that you seem to be feeling anxious about
something."
13
C. "These bruises are more consistent with spousal abuse than
accidents."
D. "You are leaving out the most important part of the story."
E. "You are lucky that you do not have children because your husband
would probably abuse them, too."
Explanation:
The correct answer is B. From this patient's behavior you should be able to
recognize that she is obviously anxious and upset (tearing at the tissue,
clenched fist, and watery eyes). It is important to carefully confront her with
your observations and try to encourage her to clarify the discrepancies in her
history. By saying, "I can't help but notice that you seem to be feeling
anxious about something," you are opening the door for her to try to explain
a situation that she is obviously denying.
It is inappropriate to say, "For such a smart woman, you seem to have
gotten yourself in a dangerous position" (choice A). This type of remark will
make her defensive and is actually insulting.
It is certainly possible that she obtained these bruises from accidents, but it
is also possible that she is being abused. It is very important to try to
differentiate this issue while trying to remain neutral and calm. It is
inappropriate to jump to conclusions by saying "these bruises are more
consistent with spousal abuse than accidents" (choice C).
While it seems as if she is not telling you the whole story, it is best to try to
encourage her to explain the situation by carefully confronting her, as
opposed to accusing her of "leaving out the most important part of the story"
(choice D). Making accusations will make the patient defensive and hostile.
Because you have not yet determined if she is being abused, it is incorrect
to say, "you are lucky that you do not have children because your husband
would probably abuse them too" (choice E). The patient will not appreciate
your rush to judgment. It is best to try to demonstrate understanding and
support by interpreting her behavior with a comment such as, "you seem
upset."
A 34-year-old man is postoperative day number 3 from an appendectomy. He
had presented 3 days earlier to the emergency department complaining of
epigastric pain radiating to his right side in association with an elevated white
blood cell count and mild fever. A CT scan of the abdomen revealed
appendiceal stranding and the patient was taken to the operating room for an
uneventful appendectomy under general anesthesia. His past medical history
is significant for intravenous heroin abuse for the past 8 years. He has been
hospitalized for withdrawal reactions on numerous occasions but continues to
use heroin daily. His last use of heroin was 5 days ago. Now, he continues to
have an ileus and has yet to pass flatus or stool. His temperature is 37.0 C
(98.6 F) and blood pressure is 120/80 mm Hg. He has a non-tender, nonerythematous surgical site. His medications consist of intravenous morphine
14
for pain control and diazepam for anxiety. The most likely cause of this
patient's ileus is
A. heroin use
B. infection
C. intraabdominal abscess
D. intravenous morphine
E. low-fiber diet
Explanation:
The correct answer is A. All patients that abuse intravenous narcotics are
constipated. Sometimes, these patients have such severe constipation that
they develop ulcerations of their rectal mucosa from impaction. These
people present a challenge for surgical care since all post-surgical
abdominal patients have an ileus secondary to manipulations during their
surgery and from anesthesia. For this patient, 3 days after an
appendectomy, his operative causes of ileus are all resolved and any
residual ileus is most likely secondary to preexisting conditions. The fact that
he has not used heroin for days is not relevant since the impacted material
in his colon will not be expelled even with increased colonic motility.
Generalized infection (choice B) is a cause of postoperative ileus, but there
is no evidence from the physical examination or from the patient history that
suggests such a process, the same is true for a localized intraabdominal
abscess (choice C). These patients almost universally manifest fever and an
elevated white blood cell count.
Although intravenous morphine (choice D) can certainly be responsible for
ileus, especially in non-sensitized patients, in this case, the morphine, when
compared to the heroin, is not a concern. This patient is highly tolerant to
opiates and therefore the morphine will have no effect.
A low-fiber diet (choice E) is not a cause of postoperative ileus but it is a
cause of constipation.
A 4-year-old girl is brought to the emergency department by her babysitter
because of "bumps on her vagina." The babysitter was giving the girl a bath
when she noticed the lesions and was so concerned that she brought her to
the hospital without calling the girl's parents. The patient is very withdrawn
and will not speak directly to you. The babysitter, who is 13 years old,
whispers to you that the girl's father has made "inappropriate passes"
towards her in the past and the girl has told her that her father touches her
and "does other things to her." Her grandparents live in the apartment directly
next door. Physical examination shows multiple vesicles on her vagina and a
frothy vaginal discharge. A wet mount of the discharge shows motile
flagellated organisms. You administer a dose of metronidazole and call her
15
house. Nobody is home in the house, but they have an answering machine.
The most appropriate next step is to
A. call the patient's grandparents and have them come pick her up
B. contact the state child protective services agency
C. leave a message on the family's machine saying that you would
like to talk about "what they do to their daughter"
D. prescribe metronidazole and acyclovir and send her home with the
babysitter with written instructions to give to her parents
E. keep her in the emergency department until her parents arrive and
she can be discharged to them
Explanation:
The correct answer is B. This patient most likely is a victim of sexual abuse,
given that she has trichomonas and may have herpes and that she told her
babysitter that her father touches her and "does other things to her."
Physicians MUST report abuse to the state child protective services agency.
This patient should be admitted to the hospital for treatment and protection.
It is inappropriate to call the patient's grandparents and have them come
pick her up (choice A) because she needs to be protected at this time while
a full evaluation of the situation is done.
While you should try to call the patient's parents, you should not leave a
message on the family's machine saying that you would like to talk about
"what they do to their daughter" (choice C). You should not accuse them
directly and you should especially not do it on an answering machine.
It is wrong to prescribe metronidazole and acyclovir and send her home with
the babysitter with written instructions to give to her parents (choice D)
because this should be reported to the state child protective services agency
and she should be admitted to the hospital for treatment and protection.
Keeping her in the emergency department until her parents arrive and she
can be discharged to them (choice E) is wrong because this patient is most
likely a victim of sexual abuse and needs to be admitted to the hospital for
treatment and protection. This case must also be reported to the state child
protective services agency. Discharging her to her parents is wrong because
she has told the babysitter that her father is the abuser.
A 6-month-old infant is brought to the emergency department by his mother
because of a 3-hour history of vomiting, abdominal distention, and general
"crankiness". She asks to see the surgical resident on call because she
"knows that her baby has an obstruction." She tells you that he has had 3
previous surgeries at 3 different hospital within a 60 mile radius of your
hospital, the first at 1 month of age for a volvulus, and the next 2 were
exploratory laparoscopies with lysis of adhesions at 3 months and 4 months
of age. His temperature is 37 C (98. 6 F), blood pressure is 90/60 mm Hg,
16
pulse is 130/min, and respirations are 30/min. His abdomen is soft with
normal, active bowel sounds. He does not wince or move as you palpate his
abdomen. Rectal examination shows soft, brown stool that is guaiac
negative. He is laughing and smiling as you perform the examination. You
ask the mother why she came to this particular hospital tonight and she says,
"they don't know what they are doing in those other places." She then asks
you if you paged the surgical resident. The most appropriate next step is to
A. admit him, page the surgery resident, and prepare him for
immediate surgery
B. draw blood for electrolytes and a complete blood count
C. order a barium enema
D. order a supine and upright radiograph of the abdomen
E. try to obtain a more detailed history from the mother and physicians
at the other hospitals
Explanation:
The correct answer is E. This seems to be a case of Munchausen's
syndrome by proxy, which is a factitious disorder characterized by the
intentional production or feigning of symptoms in order to assume the sick
role. The mother is the one who is producing or making up the symptoms for
her infant son, who is unfortunately the one suffering the consequences from
unnecessary procedures. He may have truly had a volvulus initially, and the
first surgeries may have been warranted, however at this time it seems
necessary to look into this further, before subjecting this apparently healthy
child to more tests.
Admitting him and preparing him for surgery (choice A) is inappropriate
because he appears healthy and it seems that this may be a case of
Munchausen's syndrome by proxy.
Drawing blood for electrolytes and a complete blood count (choice B) would
be correct if you thought that this patient was sick, however he appears
healthy.
A barium enema (choice C) and a supine and upright radiograph of the
abdomen (choice D) are not indicated at this time because he appears
completely fine, even though the mother tells a different story. These may
possibly be necessary at a later time, after you look into the possibility of
Munchausen's syndrome by proxy.
A 49-year-old Caucasian woman with a history of bipolar disorder type I has
come to the clinic because of increased fatigue, forgetfulness, and poor
concentration in the past couple of weeks. She complains that it takes her
forever to clean the house, and that she doesn't feel as efficient as usual. Her
daughter criticizes her weight gain, even though the patient denies increased
appetite. She denies any recent mood swings, and admits to being compliant
with her prescribed lithium. She is worried that she is getting Alzheimer's
17
dementia, because her paternal grandmother suffered from it. Her daughter is
worried that the mother is getting depressed, even though this depression
seems different from the previous episodes she has had in the past. Her
physical examination, aside from mild obesity and mild lower extremities
edema, is otherwise unremarkable. Her lithium level is 1.1mEq and complete
blood count and basic metabolic profile are within normal limits. The most
appropriate next step is to
A. begin treatment with selective serotonin reuptake inhibitor
B. discontinue lithium
C. order a dexamethasone suppression test
D. order thyroid stimulating hormone level
E. schedule a brain MRI
Explanation:
The correct answer is D. The most reasonable step in the management of
this patient is to order serum TSH. The symptoms that the patient describes
can be a consequence of hypothyroidism. Long-term lithium treatment can
cause iatrogenic hypothyroidism. The psychiatric manifestations of
hypothyroidism include fatigue, depression, lethargy, psychomotor
retardation, poor concentration, and forgetfulness. If a patient has been
otherwise stable on lithium for years, there is no need to switch to another
mood stabilizer, but rather, add substitutional therapy with levothyroxine .
Beginning treatment with an SSRI (choice A) would need to be justified after
other medical conditions that can present with depressive like symptoms are
ruled out. One would also need to be careful about the lithium-SSRI
interactions and the possibility of lithium toxicity.
Discontinuing lithium (choice B) is an inappropriate choice because the
patient has a diagnosis of bipolar I disorder and has been stable on the drug
of choice for that disorder, which is lithium. Only, if other severe side effects
from lithium were to be detected, would switching to another mood stabilizer
be justified.
A dexamethasone suppression test (choice C) is not used as a routine test
for diagnosis of psychiatric conditions. It has been used in the past to help
confirm major depressive disorder since severely depressed patients show
nonsupression of cortisol after the administration of dexamethasone. Since
the sensitivity of the test is only 45% and there are a variety of medical
conditions and pharmacological agents that may interfere with its results, it
is used nowadays infrequently for research purposes.
Scheduling a brain MRI (choice E) should be considered only if cognitive
disturbances secondary to other medical conditions or depression are ruled
out or if there is significant neurological findings that could justify the
procedure. It is not the first step in management in this case.
18
A 58-year-old man comes to the office 7 weeks after his wife died from
complications during breast cancer surgery. He says that he "misses her like
crazy" and it is so hard for him because he did not expect to "ever lose her."
He often finds himself crying in the bathroom at work. However, it is the
weekends that are especially difficult. He goes out for dinner and to the
movies with their 2 children and his friends, but he typically feels a little
detached. It is "really hard to handle" seeing other men with their wives. He is
very "sad" and wants to know if he is going to be "okay". The most
appropriate response to this patient is
A. "It seems like you are experiencing a major depressive episode that
we can treat with fluoxetine"
B. "Let's see how you feel in a few weeks and we will discuss the most
appropriate treatment then"
C. "You are experiencing grief, which is a completely normal and
expected reaction to the loss of your wife"
D. "You have an adjustment disorder that should be treated with
psychotherapy"
E. "You should have your friends set you up with a widowed woman
who will understand you"
Explanation:
The correct answer is C. This patient is most likely experiencing a normal
and expected grief reaction to the loss of his wife. It is normal to have
intense feelings of loss, a decreased enjoyment of activities, frequent crying
spells, and a "sad", depressed mood. Minor dysfunction may occur, but it is
not pronounced or persistent. It often diminishes with time and is not
associated with suicidal ideation or psychotic symptoms.
It is inappropriate to say, "It seems like you are experiencing a major
depressive episode that we can treat with fluoxetine" (choice A) because he
is experiencing normal grief. A major depressive episode would be the
diagnosis if he had marked functional impairment, psychomotor retardation,
a preoccupation with worthlessness, suicidal ideation, or psychotic
symptoms.
It is inappropriate to say, "Let's see how you feel in a few weeks and we will
discuss the most appropriate treatment then" (choice B) because he needs
reassurance right now, not in few weeks.
"You have an adjustment disorder that should be treated with
psychotherapy" (choice D) is wrong because he is most likely having a
normal grief reaction, not an adjustment disorder. An adjustment disorder is
an abnormal, maladaptive response to a stressor that leads to significant
functional impairment and extreme sadness and stress. It is similar to
depression, but it does not meet the criteria for a major depressive episode.
The patient in this case is "sad" and has normal feelings of loss. He is able
to go to work and go out with friends and family and therefore, does not
have an adjustment disorder.
19
You should not say to him, "You should have your friends set you up with a
widowed woman who will understand you" (choice E). It is not the
physician's place to give advice as to when he should start dating again.
That should happen when he is ready. Support groups, family, and friends
may be helpful during these difficult times.
You are called to meet with a patient's family on the oncology floor. The
patient is a 60-year-old Caucasian man who is on the board of trustees of the
hospital where you are employed. His family describes the patient as an
extremely controlling and arrogant, yet emotionally vulnerable person. The
results of a recent metastatic work up indicate that the patient has
disseminated colon cancer and he has been told that the chances for 'cure' of
his cancer are 1-5%. The patient's family requests that, even if the patient
asks, he not be told of the results of his testing as it will only "depress him".
On the patient's previous mental status examination, he is found to be fully
oriented, meticulously groomed, and well related. His attitude is noted to be
dismissive; he frequently asks why you are assessing him without a surgeon
present and writes down your name for credentials verification. There is no
evidence of any psychotic process, and the patient laughs when asked about
depressive symptoms or suicidal ideation. During his hospital course, he has
frequently asked that he be kept fully informed of all pertinent details of his
medical testing. The most appropriate response to give to the patient's family
regarding their request would be that you
A. are going to comply with the patient's wishes given his rights as a
competent person with self determination of care
B. feel that, given the patient's personality, chemotherapy should be
initiated promptly with the family consenting in lieu of the patient
C. recognize the personality traits described by the family in your
interaction with the patient and will comply with their request
D. will begin a titration of antidepressant medication for approximately
1 week prior to revealing the patient's prognosis to him
E. will discharge the patient imminently given that nothing acute needs
to be done and you will let the patient's family divulge whatever they feel
is appropriate
Explanation:
The correct answer is A. The patient described does have personality traits
that suggest a possible narcissistic personality disorder. He is extremely
haughty during interaction with someone he feels does not have sufficient
training to take care of a man of his importance, is described as arrogant by
his family, and seems to make demands for special treatment while
hospitalized. However, the patient clearly demonstrates an ability to
participate in decisions pertaining to his health. He is not psychotic, nor
gives any evidence of dementing illness or severe depression. Therefore, if
his request is to know exact details about his diagnosis and prognosis, then
20
that wish must be honored.
Telling them that you feel that, given the patient's personality, chemotherapy
should be initiated promptly with the family consenting in lieu of the patient
(choice B) is also incorrect due to the patient's competence and rights to
decide appropriate treatment for himself.
Telling them that you recognize the personality traits described by the family
in your interaction with the patient and will comply with their request (choice
C) is incorrect due to the fact that the patient is competent and should
therefore be allowed to know his diagnosis, treatment options, and
prognosis.
Telling them that you will begin a titration of antidepressant medication for
approximately 1 week prior to revealing the patient's prognosis to him
(choice D) is incorrect for two primary reasons. The first is the patient's
obvious competence. The second is that the patient does not appear to be
clinically depressed, and even if he were, the patient has the right to decide
whether he receives pharmacologic treatment for depression.
Telling them that you will discharge the patient imminently given that nothing
acute needs to be done and you will let the patient's family divulge whatever
they feel is appropriate (choice E) is incorrect due to the fact that the patient
is competent and has asked that his condition be explained to him in detail
by a medical professional.
A 37-year-old woman with a history of bipolar disorder is brought to the
emergency department by ambulance obtunded and ataxic. The patient has
not been hospitalized in over 5 years and has been stably maintained on
outpatient therapy with a regimen of lithium carbonate 600 mg by mouth twice
a day. While you are examining her, she has a tonic-clonic seizure lasting
approximately 30 seconds. As part of the laboratory work-up, the patient's
lithium level is found to be 4.2 mEq/L. After basic airway and cardiovascular
support, the most appropriate next step in this patient's management is to
A. administer a loading dose of phenytoin
B. get a renal consultation for emergent dialysis
C. order an MRI of the patient's head
D. provide conservative management including fluid and electrolyte
replacement
E. send liver function tests
Explanation:
The correct answer is B. The patient presents to the emergency department
with potentially life-threatening lithium toxicity. The treatment of choice for
lithium levels greater than 4.0 mEq/L is hemodialysis.
21
A loading dose of phenytoin (choice A) is not indicated after one generalized
seizure when the patient's metabolic encephalopathy can be treated with
dialysis.
An MRI of the patient's head (choice C) may be an acceptable follow-up
study after the patient receives dialysis, especially if after dialysis treatment
the patient continues to have seizures.
For lower levels of lithium toxicity, fluid and electrolyte replacement (choice
D) may be sufficient treatment.
Hepatic toxicity, as demonstrated by liver function tests (choice E) is not
suggested by the patient's presentation in the presence of a lithium level of
greater than 4.0 mEq/L given lithium's exclusively renal metabolism.
A 14-year-old boy is brought to the office because he has been "out of
control" at home and in school for the past 6 months. The mother states that
she can no longer handle his "argumentative" and "annoying" behavior. He
refuses to do any of his assigned household chores, and he always replies to
her requests with "hostile and angry" remarks. His teacher called their house
last week to tell her that he loses his temper in class when annoyed, and
frequently blames his other classmates for mistakes that she knows that he
made. Everybody is frustrated with his behavior and they are concerned that
he is only "getting worse." A few years ago they were worried that he had a
learning disability, however tests failed to show any abnormalities. There
have not been any recent changes at home or in school. The most likely
diagnosis is
A. attention-deficit hyperactivity disorder
B. bipolar disorder
C. conduct disorder
D. depression
E. oppositional defiant disorder
Explanation:
The correct answer is E. This patient most likely has oppositional defiant
disorder, which is a pattern of negative, hostile, defiant behavior that is
characterized by frequent arguments with adults, refusing adult's requests,
deliberately annoying others, and blaming others for his mistakes. These
individuals frequently lose their temper, and they are often spiteful,
vindictive, angry, resentful, and easily annoyed. This disorder is less severe
than conduct disorder.
Attention-deficit hyperactivity disorder (choice A) is a pattern of behavior that
is characterized by inattention, hyperactivity, and impulsiveness. These
individuals are easily distracted, have difficulty staying seated, finishing
22
tasks, and following instructions, appear not to listen, often lose things, and
avoid tasks with sustained mental effort. They are fidgety, have impulsive
speech, have difficulty waiting in lines or for the teacher to call on them, and
seem to be "driven by a motor". This description does not fit with the boy in
this case who is hostile, angry, and argumentative.
Bipolar disorder (choice B) is characterized by manic or hypomanic episodes
and periods of depression. During a manic episode, individuals have a
persistently elevated, expansive, or irritable mood and often have inflated
self-esteem, have racing thoughts, participate in increased goal-oriented
activity and pleasurable activity with painful consequences. They have a
marked impairment in social and occupational functioning. Hypomania is a
less severe form of mania.
Conduct disorder (choice C) is a pattern of behavior characterized by the
persistent violation of societal rules and the basic rights of others. These
individuals are aggressive towards animals, threaten and fight with other
people, use weapons, start fires, destroy property, lie, cheat, steal, run away
from home, skip school, and stay out late at night. It is a more severe
disorder than oppositional defiant disorder, which is the diagnosis for the boy
in this case.
Depression (choice D) is characterized by a sad mood, diminished pleasure,
weight loss, insomnia, psychomotor retardation, fatigue, feelings of guilt,
worthlessness, and suicidal ideation. The patient in this case is not
experiencing any of these symptoms.
A 17-year-old girl is brought into your clinic by her mother, who is concerned
that "she really just doesn't look well to me." Her medical history is
unremarkable except for irregular menses since menarche at age 16. She
has no surgical history, does not take any medications, and has no known
drug allergies. You ask her mother to leave the room and ask the patient
about sexual activity, tobacco, drug, and alcohol use, which she denies.
There are no problems at home. She is in her senior year of high school and
is doing very well, participating in track and field as well as maintaining a 3.9
average. She is planning to attend college and hopes to become a lawyer.
Review of systems is negative except for weight loss, but the patient states, "I
have always been a little too heavy. This last year I just lost my baby fat,
that's all." She acknowledges running several miles a day as part of her
training regimen for the track team. The patient denies any problem with
eating, although she feels as though "I could still stand to lose a few pounds.
My face is way too chubby." When you speak alone with her mother, you
elicit that the patient is very strict about what foods she considers appropriate
to eat and insists on preparing her own meals. Her temperature is 37.0 C
(98.6 F), blood pressure is 105/68 mm Hg, pulse is 59/min, and respirations
are 12/min. She weighs 44.5 kg (98 lb) and is 5'5". The body-mass index
(BMI) calculates to 16 kg/m2. She is an alert, cooperative, thin young female
in no distress. Her hair appears dry and brittle. However, the remainder of the
physical examination is unremarkable. Laboratory studies, including
23
biochemical profile, erythrocyte sedimentation rate (ESR,) complete blood
count, and thyroid studies are within normal limits. The most appropriate next
step in the management of this patient is to
A. begin treatment with megestrol (Megace) to stimulate appetite
B. begin treatment with oral contraceptives to regulate her menses
C. enroll her in a multidisciplinary treatment program for eating
disorders
D. immediately admit her to the hospital on psychiatric hold since she
is a danger to herself
E. refer her to an endocrinologist to treat hyperthyroidism
F. refer her to an oncologist to rule out occult malignancy
Explanation:
The correct answer is C. This patient has many historical and physical exam
findings that are consistent with anorexia nervosa. Most patients will not
readily discuss an eating disorder with a primary physician; therefore the
index of suspicion must always remain very high. Some of the historical
signs include an altered body image, strict control of dietary intake,
oligomenorrhea, and frequent exercise. Most patients, though not all, are
female and are high achievers. Physical findings in this case include dry
brittle hair and slight bradycardia. Other findings might include hypotension,
cold and mottled extremities, lanugo hair, syncope, and osteopenia or
fractures. Although there are other reasons for weight loss, which must be
kept in the differential diagnosis, these conditions do not generally have an
overriding preoccupation with body image. It is also important to recognize
signs and symptoms of bulimia, since both eating disorders often coexist.
Treatment is aimed at restoring normal weight. This is often best
accomplished by a multidisciplinary team consisting of a physician,
nutritionist, mental health worker, and a nurse. Resumption of normal
menses is a very good indicator that an appropriate weight has been
achieved.
Use of megestrol (choice A) is not correct because the underlying disorder is
not one of decreased appetite. This medication is sometimes used as an
appetite stimulant for patients with weight loss secondary to malignancy or
AIDS. However, it is not appropriate in this setting.
Treatment of oligomenorrhea with oral contraceptives (choice B), is incorrect
because this also does not address the patient's main problem. Although
she does have irregular menses, this is most likely the result of her anorexia
rather than the cause of her symptoms. At this stage, it would be more
beneficial to address the main diagnosis.
Admission to the hospital (choice D), is not the most appropriate next step.
Indications for hospitalization include: weight which is 30% below normal for
age and development, continued weight loss despite treatment, rapid weight
loss (over 3 months), cardiac arrhythmias, systolic blood pressure less than
24
70 mm Hg, heart rate slower than 50-60 bpm, hypothermia, suicidal ideation
or intent, electrolyte disturbances, or significant dehydration. The young
woman in this case may be initially treated as an outpatient.
An endocrinologist to treat hyperthyroidism (choice E) is not correct because
she has no findings consistent with hyperthyroidism.
Referral to an oncologist (choice F) should be considered if there were there
were any findings on history, physical exam, or laboratory and imaging
studies to suggest malignancy. At this point, however, this patient's case
does not warrant referral.
A 28-year-old woman is brought to the office by her husband who says that
the police were called to their house last night because she "brandished a
knife at the neighbor." He says that his wife has been "pretty crazy" for the
past 8 months. It all started with her "believing that their neighbor's son, who
has been dead for years, has been communicating with her through the
shower faucet, and telling her to kill his parents." At first, he thought that this
was just a joke, but then he began to realize that something had to be "off"
because she began to see faces of other dead acquaintances in the
refrigerator. She stopped her daily showers, started wearing dirty cloths from
the hamper, and cut off all communication with friends and relatives. She was
constantly so disheveled and "out there" that she was fired from her job as a
grocery store clerk. Approximately 5 months ago, there was a 3-week period
where she had difficulty sleeping, a 15-pound weight loss, and there was a
generalized "sadness about her." During this period, she repeatedly
expressed feelings of worthlessness, and rarely got out of bed to watch
television, which up until then had been her most exciting activity. The
husband says that she has recently begun to complain about sleeping
"troubles" again, and she has had difficulty getting out of bed. She now says
that she "needs to get out of the office to kill those people once and for all."
She does not take any medications, use any drugs, or drink alcohol. The
most likely diagnosis is
A. brief psychotic disorder
B. delusional disorder
C. major depressive disorder
D. schizoaffective disorder
E. schizophrenia
F. schizophreniform disorder
Explanation:
The correct answer is D. This patient most likely has schizoaffective
disorder, which is characterized by a mood disorder and separate psychotic
symptoms. The individual must have at least 2 weeks of delusions or
25
hallucinations in the absence of prominent mood symptoms. Mania or
depression may be present. This patient has hallucinations, delusions, and
disorganized behavior, which are suggestive of schizophrenia. However, the
presence of the depressive symptoms makes schizoaffective disorder the
diagnosis.
A brief psychotic disorder (choice A) is characterized by delusions,
hallucinations, disorganized speech and behavior for more than 1 day but
less than 1 month. This patient's symptoms have been present for longer
than a month and she has intermittent and prominent depressive symptoms.
Delusional disorder (choice B) is the presence of non-bizarre delusions for
more than a month. The belief that one is being followed is a common
symptom. The patient in this case is having bizarre delusions.
Major depressive disorder (choice C) is characterized by a major depressive
episode, which is associated with a depressed mood, sleep disturbances,
feelings of guilt, fatigue, a change in appetite, psychomotor agitation or
retardation, and suicidal thoughts. Psychotic features may be present.
However, they resolve when the mood disorder has resolved. The patient in
this case has the psychotic symptoms in the absence of the depressive
symptoms.
Schizophrenia (choice E) is characterized by the presence of psychotic
symptoms and functional impairment for at least 6 months. Unusual
thoughts, perceptions, and behaviors are called positive symptoms, while
social withdrawal, a flat effect, and lack of energy are called negative
symptoms. The presence of positive and negative symptoms for 6 months in
the absence of a mood episode is schizophrenia. The patient in this case
has positive and negative symptoms, but also has major depressive
episodes, which is consistent with schizoaffective disorder.
Schizophreniform disorder (choice F) is characterized by the presence of
delusions, hallucinations, disorganized speech and behavior, without a
mood episode, that lasts between 1 month and 6 months. This patient has
had these symptoms for longer than 6 months and has had depressive
episodes.
A 36-year-old man is admitted to the hospital for acute management of his
schizophrenia. He is a homeless man that you often see hanging out around
the neighborhood. He has had multiple hospitalizations over the past 5 years
and they usually occur when he stops taking his medications. He usually
believes that his dead cousin speaks directly to him through fire hydrants and
that she tells him that he does not need to take any medication.
Unfortunately, she is the only person that he listens to. You are called to see
him because you have treated him many times in the past. When you get to
the floor, the nurse tells you that you should be careful when you enter the
room because orders for the medication have not been written yet. You hear
howling as you are talking to the nurse and when you get to his room you see
26
that he is kneeling at the window "howling at the moon." He becomes angry
and violent when you try to enter his room. You go back to the nurse station
and tell her to give him an injection of haloperidol and diazepam. In addition,
at this time you should
A. begin psychosocial treatment with behavior skills training
B. give dantrolene to prevent neuroleptic malignant syndrome
C. prescribe benztropine to prevent parkinsonian-like symptoms
D. prescribe clozapine to treat his negative symptoms
E. schedule immediate electroconvulsive therapy
Explanation:
The correct answer is C. In acute psychiatric emergencies, a neuroleptic
agent (haloperidol) and a benzodiazepine (diazepam) are typically given to
control the patient and aid in sedation. An anticholinergic agent, such as
benztropine, should be added to prevent parkinsonian-like symptoms
(rigidity and akinesia) that may occur in patients treated with high-potency
antipsychotic agents (haloperidol).
In this acute situation, it is inappropriate to begin psychosocial treatment
with behavior skills training (choice A). Psychosocial treatment, including
behavior skills training, multi-family groups, vocational training, and
workshops, is very important in the long-term management of schizophrenia.
During the patient's hospitalization, after the patient is stabilized, the
treatment plan should focus on practical issues, and set the stage for
outpatient psychosocial issues.
Dantrolene is the treatment for neuroleptic malignant syndrome (choice B),
which may be caused by high-potency antipsychotic agents (haloperidol). It
is not routinely given to prevent this condition. NMS is associated with a high
fever, autonomic instability, rigidity, behavioral changes, and laboratory
abnormalities such as elevated white blood cell count, creatine kinase, and
abnormal liver function tests.
Clozapine (choice D) is used as a second-line antipsychotic agent for
patients who do not respond to the typical antipsychotic medications and
have prominent negative symptoms (flat affect, poverty of speech, and
asociality). It is not typically the first agent given in an acute psychiatric
emergency. It is associated with agranulocytosis (1%) and requires weekly
monitoring of the white blood cell count.
Electroconvulsive therapy (choice E) may be used in cases of nonresponsive catatonia. It is not often used to treat an acute psychiatric
emergency with a wild and out of control patient.
A 25-year-old Caucasian woman with no past medical history presents to the
emergency department for the fifth time with a complaint of chest palpitations,
27
shortness of breath, distal paresthesias, and nausea. A full metabolic and
cardiac workup is unremarkable. Urine toxicology is also negative. You
diagnose this patient as having a panic attack. Appropriate pharmacotherapy
for this patient may include
A. haloperidol
B. lithium
C. naltrexone
D. paroxetine
E. valproic acid
Explanation:
The correct answer is D. Selective serotonin reuptake inhibitors in the higher
dosing range are the treatment of choice for panic disorder.
Benzodiazepines may also be used in the initial phases of treatment while
selective serotonin reuptake inhibitors are being titrated.
Antipsychotics such as haloperidol (choice A) have no demonstrated
efficacy in the treatment of panic attacks.
Lithium (choice B) may be used as a mood stabilizer in patients with bipolar
disorder or as an augmentation strategy for unipolar depression. It has no
indication in the treatment of panic disorder.
Naltrexone (choice C) is an opiate antagonist that has a specific use in the
treatment of opiate and alcohol dependence. It cannot be used as antipanic
prophylaxis.
Valproic acid (choice E) may also be used as a mood stabilizer in patients
with bipolar disorder and may be more effective than lithium for patients with
rapid cycling bipolar disorder. It has no indication in the treatment of panic
disorder.
A 74-year-old man is admitted to the hospital for radiation treatment for
malignant melanoma of the eye that was diagnosed 7 months ago. The
treatment is performed early in the day and you are called to see him late that
night because of a sudden deterioration in his mental status. When you
arrive, he does not recognize you, even though you have been taking care of
him for many years, and he repeatedly asks where he is. His speech pattern
is disorganized and rambling. He is normally a highly functional elderly man
who lives alone and volunteers at the local nursing home. His temperature is
37 C (98.6 F), blood pressure is 110/70 mm Hg, pulse is 70/min, and
respirations are 16/min. He is uncooperative, but a modified physical
examination is unremarkable. Mental status examination is not possible
because he has an altered level of consciousness. Laboratory studies show
no abnormalities. The most likely explanation for his symptoms is that
28
A. although he appeared normal before he was admitted to the
hospital, he now has a delusional disorder
B. he already had a low level of dementia prior to hospitalization that
is being exacerbated by psychologic and physical stress
C. he is experiencing an adjustment disorder due to the recent
diagnosis of melanoma
D. he is having a brief psychotic disorder from the radiation treatment
E. he is in an acute confusional state from the psychologic and
physical stress of this hospital experience
Explanation:
The correct answer is E. This patient has delirium, which is also sometimes
called acute confusional state. It is very common in hospitalized and
institutionalized elderly individuals. It is characterized by a rapid onset of
impaired cognition, altered level of consciousness, disturbances in attention
and psychomotor activity, and altered sleep-wake cycles. The symptoms
tend to fluctuate and it is usually reversible when the underlying disorder is
identified and treated. Common causes include psychologic and physical
stress (for example, surgery), metabolic disturbances, neoplasms,
infections, medications, cerebral and cardiovascular diseases, and
withdrawal from alcohol and prescription medications.
Delusional disorder (choice A) is characterized by the presence of
nonbizarre delusions that last for more than 1 month. Functioning is not
usually impaired. Antipsychotic agents and psychotherapy may be
necessary. This patient is not delusional. He is in an acute confusional state.
Dementia (choice B) is the slow and insidious onset of cognitive and
intellectual deficits with no changes in consciousness. The symptoms are
stable and irreversible. The most common causes are Alzheimer disease
and multi-infarct dementia. There is no evidence in this case that he had
dementia prior to this hospital admission. Also, the case states that he has
an altered level of consciousness, which is typically associated with delirium,
not dementia.
Adjustment disorder (choice C) is a maladaptive response to a stressful
event. The symptoms include mood disturbances, behavioral changes, and
impaired functioning. It usually resolves within 6 months. Since he was
diagnosed with melanoma 7 months ago, it is unlikely that this acute
confusional state is due to an adjustment disorder. Delirium is a more likely
diagnosis.
Brief psychotic disorder (choice D) is the abrupt onset of psychotic
symptoms including hallucinations, delusions, and disorganization with
impaired functioning. It is present for more than 1 day, but less than 1
month. It is usually preceded by a stressful life event. This patient has been
experiencing these symptoms for less than a day, and therefore he cannot
be diagnosed with a brief psychotic disorder.
29
A 71-year-old retired police officer comes to the clinic for his annual physical
examination. He has no complaints, but you notice that he is not his usual
self. He appears distraught and distant, speaks in a low voice, and avoids
eye contact. He seems to have lost some weight, and admits to not caring
about his meals. The physical examination and routine laboratory tests are
unremarkable. On further questioning the patient shares that his wife had
died several months ago. He lives alone, has no children, and misses her a
lot. You ask him about his daily activities. It seems that he stays to himself
and rarely sees his fellow officers or friends any more. The most important
next step is evaluation for
A. Alzheimer's dementia
B. feelings of guilt
C. religious preference
D. suicidal ideation
E. weight loss
Explanation:
The correct answer is D. It is important to ask this patient openly about
suicidal ideation, plans, or an existing wish not to live anymore. Apart from
anhedonia and other signs of depression or complicated grief, he seems to
have all the risk factors for suicide present. These risk factors include being
male, older, retired, widowed, having potential access to weapons since he
was a police officer, and having no close social support.
Alzheimer's dementia (choice A) in an elderly patient who seems to have
some cognitive difficulties, should be considered. However, in this case,
pseudodementia secondary to depression may be present. The imminent
risk of a possible suicide attempt is, in this case, more important to assess. If
that is ruled out, other tests and steps can be taken to rule out other medical
conditions.
Guilt feelings (choice B) may be a part of the depressive syndrome or grief,
if a patient feels that he should have been the one that deserved to die. Guilt
feelings can sometimes also lead to suicidal ideation. However, it is
important to ask about suicidal ideation independently before continuing
further assessment.
Religious preference (choice C) can be explored in light of the possible
suicidal ideation. If a patient is practicing his religion, even though suicidal
ideation may be present, the fact that most religious beliefs stand against
self-harm, may be helpful in preventing suicide. However, exploring this
comes only after the presence of suicidal ideation is established.
Weight loss (choice E) in light of other normal findings is not the most urgent
concern. It should be considered as a part of the depressive syndrome and
monitored.
30
A 16-year-old girl comes to the office after her boyfriend of 2 years abruptly
ended their relationship. She has been a patient of yours for the past 7 years
and seems to feel very comfortable talking about relationships and sexual
issues with you. She says that she just decided to have sexual intercourse
with him a month ago and that this was a "huge deal" because it was her first
time. Now, after "trusting him" with such a "personal thing”, he has
"abandoned" her. She suddenly becomes silent and begins to cry. She takes
a tissue out her pocketbook and remains silent. At this point you should
A. advise her to "pull herself together"
B. maintain eye contact, and after a few minutes say, "I understand
that this is hard for you"
C. remain silent for however long she needs to compose herself
D. tell her not to cry and say, "he is not worth getting so upset over"
E. use this time to review the notes in her chart
F. recommend that she go talk to a psychiatrist
Explanation:
The correct answer is B. Silence during a patient interview can be
uncomfortable for both the patient and the physician. It is important to use
this time to show that you are supportive and attentive. You can hand her a
box of tissues if she does not have any and maintain eye contact to show
that you are "still with her" (not dozing off thinking about other things). After a
little while (a couple of minutes) you should say something to show your
support and understanding and to try to facilitate the conversation. "I
understand that this is hard for you" is perfect for this time.
Advising her to "pull herself together" (choice A), and telling her not to cry,
and that "he is not worth getting so upset over" (choice D) is incorrect
because this only implies that it is not "okay" to show emotions and that she
is being immature or foolish.
While it is important to remain silent for a few minutes to allow her to release
tension by crying, it is not realistic to remain silent for however long she
needs to compose herself (choice C). You probably have other patients
waiting to be seen, so you need to give her some time, but not however long
she needs. Leaning forward, maintaining eye contact, and after a few
minutes making a supportive comment is the most appropriate way to
manage this situation.
Using this time to review the notes in her chart (choice E) might seem like a
good idea, but it just makes it seem like you are not "there for her"
emotionally and that she is wasting time that can be better spent "reviewing
notes." You need to seem supportive (eye contact) and make her feel
comfortable. Reviewing notes at this time will only make you seem aloof.
31
Recommending that she go talk to a psychiatrist (choice F) is unnecessary
at this time. She is willing to discuss these issues with you and so you
should be helpful and supportive. A psychiatrist may be appropriate in the
future if she has any thoughts of suicide or wants to discuss these issues at
great lengths.
A 78-year-old African American woman with no past psychiatric history is
admitted to the inpatient geriatric floor with urosepsis. Prior to hospitalization
she lived in the community by herself and was able to tend to her own
cooking, cleaning, and shopping. On the second night in the hospital, the
nursing staff alerts you that the patient is extremely agitated and combative.
She had pulled out her intravenous lines and appears to be speaking to
people who are not in the hospital room. After placing her in soft restraints,
the most appropriate pharmacologic treatment for behavioral management of
this patient may include
A. diazepam
B. electroconvulsive therapy
C. fluoxetine
D. haloperidol
E. lithium
Explanation:
The correct answer is D. Low doses of high potency antipsychotics such as
haloperidol are the treatment of choice for agitation in the context of
delirium. In this patient, haloperidol may be used in conjunction with
environmental measures such as dim light in the room, soft restraints, and a
bedside companion.
Diazepam (choice A) would not be an appropriate treatment for this patient,
given the potential for disinhibition when administered to the elderly patient
and its long half-life and potential for prolonged sedation and accumulation
in adipose tissue after repeated doses.
Electroconvulsive therapy (choice B) would not be an appropriate treatment
for a patient with delirium. It is indicated in conditions such as severe
depression, depression in pregnancy, for neuroleptic malignant syndrome,
and refractory mania.
Fluoxetine (choice C) is used to treat depression. It is not used for acute
delirium.
Lithium (choice E) is not an established treatment for delirium. It does not
offer control for agitation after 1 dose and is potentially dangerous in patients
for whom dehydration may be a medical comorbidity.
32
A 33-year-old man comes to the emergency department because of a 12hour history of increasing dizziness and palpitations. He tells you that he
knows that he is "lapsing into severe hypoglycemia" because of an untreated
insulinoma. He is an intensive care nurse at a hospital in a neighboring town
but says that he does not like the doctors there so he goes to many different
area hospitals. He tells you that he has been treated for hypoglycemia on 3
separate occasions at 3 different hospitals and is too frightened to undergo
the surgery to remove the tumor. He tells you that he is otherwise healthy and
does not take any medications. His last food intake was 12 hours ago. His
temperature is 37.2 C (99.0 F), blood pressure is 130/90 mm Hg, pulse is
11/min, and respirations are 18/min. Physical examination shows diaphoresis
and pallor. His glucose level is 35 mg/dL. He has high insulin levels with a
decreased C-peptide level. You give the patient glucose-containing fluids and
try to obtain his previous medical records from your hospital but they were
destroyed in a flood. You are unable to reach any of the other doctors that
have treated him in the past. The next step in management is to
A. consult with a surgeon for possible resection of the insulinoma
B. discuss the possibility that he is producing these symptoms
C. measure blood levels of sulfonylureas
D. order a CT scan of the abdomen
E. tell him that he is malingering
Explanation:
The correct answer is B. This patient most likely has a factitious disorder
(Munchausen syndrome), which is the active production of symptoms to
assume the sick role, and so you should discuss the possibility that he is
producing these symptoms. Individuals with this disorder are often health
care workers who know how to fake symptoms. In this case, this nurse is
complaining of hypoglycemia and an insulinoma and while he does have low
glucose and high insulin levels, he has a decreased C-peptide level. These
values are consistent with exogenous insulin and not an insulinoma. Cpeptide levels are increased when there is endogenous insulin production
(insulinoma, autoimmune hypoglycemia, or sulfonylureas) because Cpeptide is produced during the processing of proinsulin.
As stated above, this patient's laboratory studies are inconsistent with an
insulinoma and so there is no reason to consult with a surgeon for possible
resection of an insulinoma (choice A).
Measuring the blood levels of sulfonylureas (choice C) would be the next
step if he had elevated C-peptide levels and you still suspected a factitious
disorder. Sulfonylureas stimulate endogenous insulin production and is
therefore associated with an elevated C-peptide. The only way to distinguish
this from autoimmune hypoglycemia and an insulinoma is to measure drug
levels.
33
A CT scan of the abdomen (choice D) is inappropriate because this patient's
laboratory studies are consistent with a factitious disorder, not an
insulinoma.
It is incorrect to tell him that he is malingering (choice E) because
malingering means that a secondary gain is present, such as financial gain
or the avoidance of unwanted duties. This patient appears to have a
factitious disorder, which is the production of symptoms to induce medical
testing and assume the sick role. There is no clear secondary gain.
A 33-year-old man comes to the emergency department because of a 12hour history of increasing dizziness and palpitations. He tells you that he
knows that he is "lapsing into severe hypoglycemia" because of an untreated
insulinoma. He is an intensive care nurse at a hospital in a neighboring town
but says that he does not like the doctors there so he goes to many different
area hospitals. He tells you that he has been treated for hypoglycemia on 3
separate occasions at 3 different hospitals and is too frightened to undergo
the surgery to remove the tumor. He tells you that he is otherwise healthy and
does not take any medications. His last food intake was 12 hours ago. His
temperature is 37.2 C (99.0 F), blood pressure is 130/90 mm Hg, pulse is
11/min, and respirations are 18/min. Physical examination shows diaphoresis
and pallor. His glucose level is 35 mg/dL. He has high insulin levels with a
decreased C-peptide level. You give the patient glucose-containing fluids and
try to obtain his previous medical records from your hospital but they were
destroyed in a flood. You are unable to reach any of the other doctors that
have treated him in the past. The next step in management is to
A. consult with a surgeon for possible resection of the insulinoma
B. discuss the possibility that he is producing these symptoms
C. measure blood levels of sulfonylureas
D. order a CT scan of the abdomen
E. tell him that he is malingering
Explanation:
The correct answer is B. This patient most likely has a factitious disorder
(Munchausen syndrome), which is the active production of symptoms to
assume the sick role, and so you should discuss the possibility that he is
producing these symptoms. Individuals with this disorder are often health
care workers who know how to fake symptoms. In this case, this nurse is
complaining of hypoglycemia and an insulinoma and while he does have low
glucose and high insulin levels, he has a decreased C-peptide level. These
values are consistent with exogenous insulin and not an insulinoma. Cpeptide levels are increased when there is endogenous insulin production
(insulinoma, autoimmune hypoglycemia, or sulfonylureas) because Cpeptide is produced during the processing of proinsulin.
34
As stated above, this patient's laboratory studies are inconsistent with an
insulinoma and so there is no reason to consult with a surgeon for possible
resection of an insulinoma (choice A).
Measuring the blood levels of sulfonylureas (choice C) would be the next
step if he had elevated C-peptide levels and you still suspected a factitious
disorder. Sulfonylureas stimulate endogenous insulin production and is
therefore associated with an elevated C-peptide. The only way to distinguish
this from autoimmune hypoglycemia and an insulinoma is to measure drug
levels.
A CT scan of the abdomen (choice D) is inappropriate because this patient's
laboratory studies are consistent with a factitious disorder, not an
insulinoma.
It is incorrect to tell him that he is malingering (choice E) because
malingering means that a secondary gain is present, such as financial gain
or the avoidance of unwanted duties. This patient appears to have a
factitious disorder, which is the production of symptoms to induce medical
testing and assume the sick role. There is no clear secondary gain.
An 81-year-old woman is admitted to the medical floor for depression and
questionable dementia. She has a past medical history of insulin-dependent
diabetes mellitus, hypertension, and an anterior wall myocardial infarction 10
years ago. She takes nifedipine, atenolol, insulin, aspirin, furosemide,
simvastatin, multivitamins, docusate sodium and was recently started on
methylphenidate for depression by her primary care physician. She lives at
home and her family reports that she has become increasingly withdrawn and
confused over the past few months.She is an obese woman in no distress
with normal vital signs. Her neck is supple with a normal thyroid, clear lungs,
and 1+ non-pitting lower extremity edema. She is alert to person, month, but
not to date or location. She has poor concentration. In addition to evaluation
of her medication list for potential causes of confusion, the most important
test to order in the evaluation of this patient's confusion is
A. calcium level
B. diffusion weighted MRI of brain
C. head CT scan with contrast
D. rapid plasma reagin test
E. thyroid stimulating hormone
Explanation:
The correct answer is E. Hypothyroidism is an uncommon cause of
confusion. However, in the elderly, especially women, hypothyroidism or
subclinical hypothyroidism is extremely common; up to 20% of all medical
inpatients in some series. Given this, the evaluation of new confusion or
35
suspected dementia should always include a screening TSH level when the
patient is of this age demographically.
Calcium level (choice A) is included as well in the initial laboratory
evaluation, but interpretation of this test is difficult. The primary reason for
this is that asymptomatic hypercalcemia is very common in the elderly, with
as much as a 50% prevalence rate. Given this, even the finding of elevated
total serum calcium is without obvious significance and hypothyroidism is still
a much more common cause of altered mental status despite its lower
prevalence.
Diffusion weighted MRI of brain (choice B) is an MRI that uses diffusion
weighting which allows visualization of areas of acute infarction. This is only
used in special circumstances when the question of an evolving infarct is
suspected.
Head CT scan with contrast (choice C) is not indicated unless there is
clinical suspicion for old infarcts or a bleed.
Rapid plasma reagin test (choice D) is the screening test for syphilis.
Although this test is routinely sent with the panel of tests for the evaluation of
dementia, tertiary syphilis is exceedingly rare and not a very common cause
of dementia in any age demographic in the United States.
A 2-year-old boy is brought to the office for the first time by both his parents
for a well-child visit. The family just moved to your city from a small town,
where they were very dissatisfied with the medical care. They tell you that the
boy has been "a little off since birth." He failed to ever develop a social smile,
his face is expressionless, and he does not make eye contact. He often plays
by himself in the corner, despite the parent's effort to make numerous playdates with other children his age. They often find him twirling the buttons on
his shirt for hours at a time. He appears to be a well-developed child,
however he refuses to speak to you. The parents are not actually sure if he is
able to speak. The most likely diagnosis is
A. attention-deficit hyperactivity disorder
B. autistic disorder
C. childhood-onset schizophrenia
D. oppositional defiant disorder
E. Tourette's disorder
Explanation:
The correct answer is B. This child most likely has autism, which is a
pervasive developmental disorder that is characterized by the impairment of
social interactions, impairment of communication, and repetitive, stereotyped
behaviors. It is more common in boys and in a sibling of a child with autism.
It is associated with mental retardation, seizures, and self-mutilation. The
36
treatment includes structured classroom training with behavioral
management techniques and haloperidol. Mood stabilizers may decrease
the self-harming behaviors.
Attention-deficit hyperactivity disorder (ADHD) (choice A) is a childhood
disorder characterized by impulsiveness, hyperactivity, easy distractibility,
and inattention. Methylphenidate, a psychostimulant, is the
pharmacotherapy of choice. Weight loss and insomnia are major side
effects. The child in this case is not overactive or inattentive.
Childhood-onset schizophrenia (choice C) may be diagnosed in a child that
develops positive and negative psychotic symptoms between 5 years of age
and adolescence or early adulthood. The children often have normal
intelligence. Childhood-onset schizophrenia is often associated with a
gradual lifelong decline in functioning and a poor response to medication.
Treatment includes antipsychotic agents and structured classroom training
with behavioral management techniques. It is unlikely that this 2-year-old
child has this disorder. He appears to have impairment of social interactions
and impairment of communication, not psychotic symptoms.
Oppositional defiant disorder (choice D) is a pattern of behavior
characterized by the unwillingness to comply with the rules of others and
majorly difficult, disruptive behavior. It is generally regarded as a less severe
form of conduct disorder. The child in this case does not fit this description.
Tourette's disorder (choice E) is characterized by motor and vocal tics.
Patients say that they try to control the behavior, but the urge is often too
overwhelming. It is associated with obsessive-compulsive disorder. The child
in this case does twirl his shirt buttons a lot, however this is not a tic, which
is a sudden, recurrent, rapid behavior.
You are called to obtain surgical consent from an 84-year-old man who is
hospitalized after suffering a right femoral neck fracture, for repair of that
fracture. The patient has a long history of multi-infarct dementia and major
depression. He lives with his daughter and her husband. They report to you
that he lost his footing while walking and suffered a fall. There was no loss of
consciousness and no evidence by history that the fall was syncopal in
nature. You read the psychiatric evaluation in his chart that notes that the
patient, despite some dementia, confusion, and odd mannerisms, is
competent to make his own medical decisions and fully understands the
nature of his condition. When you enter the room to obtain consent from the
patient for surgery, he responds with situation-inappropriate responses, fails
to make eye contact with you, and is not oriented to place or time. The patient
has no power of attorney or health care proxy listed in his medical record.
The most appropriate course of action is to
A. call a family member to consent for the patient
B. consent the patient for surgery, he has been cleared by psychiatry
37
C. consent the patient for surgery with a note explaining his condition
in the chart
D. do not consent the patient for surgery
E. perform your own competency test on this patient
Explanation:
The correct answer is D. Situations such as this frequently arise in in-patient
settings. The concept of informed consent has at its core, the requirement
that the patient is clear of mind so that they may understand the
conversation being had. Although many people argue that most patients fail
to understand complex medical issues involved in such decisions, a patient
must be at least of a mental status that allows their independent intelligence
and thinking ability to be the limiting factor.
Calling a family member to consent for the patient (choice A) is only
appropriate if that family member has been appointed as a designated
health care proxy or has legal power of attorney for the patient. Family
members cannot be used as a convenience tool when a patient has altered
mental status.
Consent the patient for surgery, he has been cleared by psychiatry (choice
B) is incorrect because irrespective of what another physician's opinion is,
the consenting physician must feel that the patient is able to understand the
conversation that they will be having.
Consent the patient for surgery with a note explaining his condition in the
chart (choice C) presupposes that the surgery being considered is
emergent. In such cases, when the patient is an adult and no advance
directive exists, the surgery may be performed regardless of mental status.
In the case of this patient, since the procedure is non-emergent, consent
must be obtained.
Performing your own competency test on this patient (choice E) is not an
option since competency is a legal term. Judges determine competence.
A 48-year-old woman with diabetes mellitus is brought by her ex-husband
and son to the emergency department following a suicide attempt by
overdosing on "some pills". She has been depressed and saw a counselor
after her husband left her for his secretary. She has had a hard time dealing
with the separation and has, according to her son, become withdrawn lately.
He knows that she was receiving some psychiatric medication from her
primary care physician because of pain related to her diabetes mellitus. She
has flushed dry skin, dilated pupils, tachycardia, and confusion. After the
bottles are checked, it seems that the patient overdosed on amytriptyline. The
family is concerned about her. They feel she needs to deal with her
psychiatric problem. The most appropriate intervention at this time is to
A. admit her to psychiatry for further treatment
B. admit her to the intensive care unit
38
C. admit her to the medical floor
D. call psychiatry to schedule an outpatient clinic appointment
E. talk to her ex-husband about the need to get marital counseling
Explanation:
The correct answer is B. This patient needs continuous monitoring because
she overdosed on a tricyclic antidepressant. This attempt can lead to cardiac
arrhythmias and death. This patient also needs close monitoring because of
her current delirious condition and suicidality. Thus, the only setting that
provides this level of care, even if it is for one day only, should be the
intensive care unit. The levels of amytriptyline should be monitored .
Admission to psychiatry (choice A) is a reasonable consideration once the
patient is medically stable. This decision should be left up to the psychiatry
consult team after they have fully evaluated the patient.
Admission to regular medical floor (choice C) is not an immediate option. It
can be done after the patient has been stabilized in the intensive care unit,
and the need for less intense monitoring is established. However, given the
patient's serious suicide attempt, suicide precautions should be placed, until
the psychiatrist's final decision.
An outpatient psychiatry appointment (choice D) is currently not an option
since the patient has precipitating events that lead to this suicide attempt. It
seems that she currently lacks enough coping skills to handle separation in
a more appropriate way. It may be placed later after the acute and critical
issues are addressed.
Talking to ex-husband about marital counseling (choice E) is not an
appropriate thing to do in this situation. Even if there was a possibility of
reconciliation, this should be done in therapy on an outpatient basis, by
people trained to specifically provide that treatment.
A 45-year-old homeless man is brought to an emergency department by
ambulance after being found in the street obtunded, combative, and agitated.
The patient is unable to provide any history in the emergency room due to his
altered mental status. His temperature is 38.1 C (100.6 F), blood pressure is
200/110 mm Hg, and pulse is 110/min. While in the emergency department,
the patient has a generalized tonic-clonic seizure lasting approximately 30
seconds. The patient's presentation is most consistent with withdrawal from
A. alprazolam
B. caffeine
C. cocaine
D. heroin
39
E. tricyclic antidepressants
Explanation:
The correct answer is A. Withdrawal from a benzodiazepine, such as
alprazolam, is a potentially fatal condition that can involve psychomotor
agitation, frank delirium, autonomic instability, and seizures. In this patient,
another consideration with a similar clinical presentation would be delirium
tremens from alcohol withdrawal. Both conditions would be treated with
supportive measures and acute parenteral benzodiazepine treatment.
Caffeine (choice B) has a potential withdrawal syndrome associated with
irritability, possible concentration difficulties, and headache. However, even
abrupt discontinuation of caffeine use is not considered to be at all
dangerous.
Cocaine withdrawal (choice C) is characterized by intense feelings of
dysphoria, loss of energy, increase in appetite, and intense cravings for the
drug. It is not usually associated with delirium, autonomic instability, or
seizures and does not carry an associated risk of fatality.
Heroin withdrawal (choice D) is characterized by piloerection, nausea, and
diarrhea and can include modest increases in blood pressure and heart rate.
However, it is not characterized by delirium, withdrawal seizures, or
autonomic instability and is not considered to carry a high risk of fatality.
Clonidine or a long-acting opiate agonists such as methadone are the
treatments of choice for withdrawal from heroin.
Tricyclic antidepressants (choice E) do not have an associated withdrawal
syndrome and may be discontinued abruptly, especially if treatmentemergent complications arise.
A 29-year-old woman who just delivered a healthy baby boy 4 days earlier
comes to the office because she "is just not feeling right." She says that she
has been very anxious and "tearful" over the past 2 days. She is not exactly
sure that she is "doing everything correctly" and she feels like she is going to
be an incompetent mother. She is on a 2-month maternity leave from her job,
but her husband is unable to take any time off. There are tears rolling down
her cheeks and her hands are trembling. At this time the most appropriate
next step is to
A. admit her to the hospital for treatment and observation
B. give her a prescription for diazepam and tell her that she can
continue to breast-feed
C. prescribe fluoxetine and advise her to discontinue breast-feeding
while on the medication
D. reassure her that this is very common and no medication is
indicated at this time
40
E. try to contact her husband and advise him that his wife currently
needs extra support and understanding
Explanation:
The correct answer is D. This patient most likely has postpartum "blues",
which is a common term used for the mild depression that more than 50% of
women experience approximately 3 days postpartum. This syndrome is
characterized by anxiety, confusion, sadness, and irritability. It is usually a
self-limited syndrome that spontaneously resolves within 2 weeks. These
patients often need reassurance, understanding, and education. It is
important for them to receive this extra support from both medical personnel,
and their family/spouse.
Admission to the hospital for treatment and observation (choice A) is not
necessary at this time for what seems like a common case of postpartum
blues. Medical, and family support, and reassurance is all that is indicated
right now. If she were to begin to display psychotic symptoms with delusions
or severe depressive symptoms, hospitalization may be necessary to treat
the symptom,s and to prevent suicide or infanticide.
It is inappropriate to give her a prescription for diazepam, and tell her that
she can continue to breast-feed (choice B), because she seems to be
experiencing postpartum blues, which is a very common, transitory disorder.
Medication is not generally indicated. Also, she should discontinue breastfeeding if she were to take diazepam.
You should not prescribe fluoxetine and advise her to discontinue breastfeeding while on the medication (choice C) at this time. It seems that this
patient has postpartum blues, which is a very common syndrome that
women experience after giving birth. It is often transitory and only requires
support and understanding. True "postpartum depression" is generally not
diagnosed until the woman experiences 2 weeks of depressed mood with
changes in sleep, guilt, hopelessness, anxiety, fear, jealousy, impaired
concentration, and other depressive symptoms. The exact cause is
unknown, but it is thought to be due to changes in hormone levels, family
history, and stress. It is correct to tell her to stop breast-feeding while taking
antidepressants.
While it is correct that this woman with postpartum "blues" needs family
support now, it is not appropriate to try to contact her husband and advise
him that his wife currently needs extra support and understanding (choice
E). This woman is your patient and you should try to educate her and
reassure her that this is very common. If the husband was in the room at the
time, you could try to explain to both of them that this is a stressful time and
that everybody needs to give extra support. Calling the husband is not the
best choice.
A 58-year-old woman is admitted to the hospital with a 2-day history of
lethargy and decreased oral intake. She is unable to give a clear history, but
her daughter who brought her to the hospital says that she lives alone and
41
she usually calls her every few days. When her mother did not answer her
phone she went to her home and found her asleep on the couch with alcohol
on her breath. She was difficult to arouse and walked with an unsteady gait.
Her mother is reluctant to go to doctors and has lived alone since her
husband died 5 years ago. Her mother has always enjoyed drinking but in the
last few years has increased the frequency and quantity. She has a history of
hypertension but is on no medications. On admission, she appears
disheveled and has a non-focal neurological examination. Laboratory studies
are normal except for an elevated alcohol level. A CT scan of the head is
normal. After 3 days of inpatient hydration and feeding, the patient is feeling
much better and is scheduled for discharge. The most important intervention
prior to her discharge is
A. abdominal pelvic CT scan to evaluate for ascites
B. health care proxy
C. liver biopsy to assess the extent of alcohol damage
D. social work consult to evaluate her living situation and arrange
referral to an alcohol abuse program
E. benzodiazepines to prevent alcohol withdrawal
Explanation:
The correct answer is D. The patient's change in mental status was most
probably due to her alcohol abuse and this issue should be addressed prior
to her discharge. The patient also lives alone which is a potentially
dangerous situation for her. The care of patients in the modern health care
environment requires a multidisciplinary approach and the ability to work
with non-physicians. Social workers can be helpful in referring patients to
substance abuse programs and in evaluating their living conditions.
At the present time there is no indication to perform a CT scan (choice A) on
this patient. On physical exam there is no mention of ascites. Evaluation of
ascites usually involves an imaging study of the abdomen along with
diagnostic paracentesis.
It is important for all adults to have a health care proxy (choice B) to
designate an individual to make decisions for them if they become
incapacitated. This is especially relevant for the chronically ill or elderly. But
this is not the most pressing issue facing this patient. Her alcoholism and
living situation should be dealt with prior to discharge and the issue of a
health care proxy can be dealt with as outpatient.
Liver biopsy (choice C) is indicated in the evaluation of unexplained liver
disease or elevated transaminases. This patient has none of the above. She
has normal LFTs and no evidence of liver disease on physical exam. She
certainly could have occult liver disease secondary to her drinking but this in
itself is not an indication for biopsy. The most important measure she could
take is to stop alcohol her alcohol consumption.
Alcohol abusers who show evidence of withdrawal should certainly receive
42
benzodiazepines to prevent delirium tremens and seizures. But this patient
has no clinical signs of withdrawal (autonomic instability or restlessness) or a
previous history of seizures when she stops drinking. In addition, she has
been stable in the hospital for 3 days and therefore at this point is unlikely to
benefit from benzodiazepines (choice E).
A 16-year-old boy is brought to the emergency department by the
paramedics, accompanied by his parents who called 911. The parents report
that their son has been acting strangely off and on for the past couple of
weeks. They say that he has been going to parties with his friends and comes
home late. He often gets up and is irritable and at times makes inappropriate
comments. When asked what he meant by the comments, he would just
move on to another topic. In the past week he has been locking himself in his
room, playing loud music, and talking loudly, even though there is no phone
in his room. This morning he started talking about the great silver cross that
can be found buried on the cemetery in the east side of town. He claimed that
he was hearing God's voice telling him to fly like a bird through the window
and get it. The parents grabbed him as he was reaching for the window. The
patient has no prior psychiatric history. Medical history is significant for an
appendectomy at the age of 11. The parents say that he was always a good
student, and basically a good, somewhat introverted, child. The parents are
confident that he is not on any drugs or prescription medication. The family
history is negative for psychiatric disorders. The patient looks tired, sleepy,
and unkempt. His blood pressure and pulse are slightly elevated, but other
than that, his physical examination is unremarkable. He is cooperative with
the examination, and oriented appropriately, but seems to be responding to
internal stimuli and unable to carry on a meaningful conversation. The most
appropriate first step in this patient's management is to
A. order a head CT
B. place the patient in 4-point restraints
C. prescribe an antipsychotic agent
D. send urine for drug screen
E. set up a psychology consult to do a Rorschach test
Explanation:
The correct answer is D. The first step in this patient's management would
be to order a urine drug screen. Even though the parents seem to think he is
not using drugs, his age, recent onset, and history of behavior in the past
several weeks may indicate possible drug abuse. Therefore, substance
induced psychotic disorder should be ruled out first.
Head CT (choice A) is a part of a workup for delirium. It also is a part of the
workup for first break psychosis. There seems to be no indication that the
patient is delirious and physical exam seems to be normal. If other
laboratory vales are unremarkable, a head CT can be done once the patient
43
is admitted to the unit.
Putting the patient in 4-point restraints (choice B) is currently not indicated.
The patient seems calm, cooperative, and not currently endangering himself
or anyone else. However, given the history of commanding hallucinations,
close observation of the patient should be in place.
Prescribing an antipsychotic (choice C) before the patient has been
medically cleared and a provisional psychiatric diagnosis established is not
justified. It should be done only in emergency situations, such as when the
patient is agitated and there is increased paranoia, hallucinatory behavior,
etc. If it is done, it should be only as a temporary measure until the final
disposition is made because of the possibility of side effects and the need to
follow up on treatment results. This patient is being cooperative, and is not
combative and so this is not immediately necessary, it can wait until a more
thorough evaluation is performed, including a drug screen.
A psychology consult to do a Rorschach test(choice E) is not necessary at
this point. Rorschach is projective test that is used to test for psychotic
material. Since this patient seems overtly psychotic and actively responds to
internal stimuli, it would be difficult and fruitless to perform this test.
A 16-year-old boy is brought to the emergency department by the
paramedics, accompanied by his parents who called 911. The parents report
that their son has been acting strangely off and on for the past couple of
weeks. They say that he has been going to parties with his friends and comes
home late. He often gets up and is irritable and at times makes inappropriate
comments. When asked what he meant by the comments, he would just
move on to another topic. In the past week he has been locking himself in his
room, playing loud music, and talking loudly, even though there is no phone
in his room. This morning he started talking about the great silver cross that
can be found buried on the cemetery in the east side of town. He claimed that
he was hearing God's voice telling him to fly like a bird through the window
and get it. The parents grabbed him as he was reaching for the window. The
patient has no prior psychiatric history. Medical history is significant for an
appendectomy at the age of 11. The parents say that he was always a good
student, and basically a good, somewhat introverted, child. The parents are
confident that he is not on any drugs or prescription medication. The family
history is negative for psychiatric disorders. The patient looks tired, sleepy,
and unkempt. His blood pressure and pulse are slightly elevated, but other
than that, his physical examination is unremarkable. He is cooperative with
the examination, and oriented appropriately, but seems to be responding to
internal stimuli and unable to carry on a meaningful conversation. The most
appropriate first step in this patient's management is to
A. order a head CT
B. place the patient in 4-point restraints
C. prescribe an antipsychotic agent
44
D. send urine for drug screen
E. set up a psychology consult to do a Rorschach test
Explanation:
The correct answer is D. The first step in this patient's management would
be to order a urine drug screen. Even though the parents seem to think he is
not using drugs, his age, recent onset, and history of behavior in the past
several weeks may indicate possible drug abuse. Therefore, substance
induced psychotic disorder should be ruled out first.
Head CT (choice A) is a part of a workup for delirium. It also is a part of the
workup for first break psychosis. There seems to be no indication that the
patient is delirious and physical exam seems to be normal. If other
laboratory vales are unremarkable, a head CT can be done once the patient
is admitted to the unit.
Putting the patient in 4-point restraints (choice B) is currently not indicated.
The patient seems calm, cooperative, and not currently endangering himself
or anyone else. However, given the history of commanding hallucinations,
close observation of the patient should be in place.
Prescribing an antipsychotic (choice C) before the patient has been
medically cleared and a provisional psychiatric diagnosis established is not
justified. It should be done only in emergency situations, such as when the
patient is agitated and there is increased paranoia, hallucinatory behavior,
etc. If it is done, it should be only as a temporary measure until the final
disposition is made because of the possibility of side effects and the need to
follow up on treatment results. This patient is being cooperative, and is not
combative and so this is not immediately necessary, it can wait until a more
thorough evaluation is performed, including a drug screen.
A psychology consult to do a Rorschach test(choice E) is not necessary at
this point. Rorschach is projective test that is used to test for psychotic
material. Since this patient seems overtly psychotic and actively responds to
internal stimuli, it would be difficult and fruitless to perform this test.
A 66-year-old woman comes to the office because of difficulty sleeping, a
decreased appetite, fatigue, an inability to concentrate, and a general "down"
mood for the past 2 weeks. She states that she and her husband are going
on a 3-week trip to Italy and France next month, and she "wants to get to the
bottom of this" before they go. She has been coming to you since her first
child was born, and over the years you have treated her for gastroenteritis,
several yeast infections, and most recently, high blood pressure. She has
always been a very compliant patient. You notice in her chart that you
prescribed propranolol for her hypertension 1 month ago, after trying to
control it with weight reduction, smoking cessation, alcohol elimination, salt
and fat reduction, and aerobic exercise. Her blood pressure at 3 previous
visits was 150/90 mm Hg and today it is 135/80 mm Hg. She does not take
45
any other medications. At this time you should
A. advise her to start a "gentle" walking routine to "lift her spirits"
B. advise her to stop taking propranolol and give her a prescription for
hydrochlorothiazide
C. explain to the patient that she is stressed about her trip, and these
symptoms will pass
D. give her a prescription for tranylcypromine
E. increase the dose of propranolol and prescribe phenelzine
Explanation:
The correct answer is B. The symptoms of change in appetite, fatigue,
insomnia, lack of concentration, and worthlessness for greater than 2 weeks
are consistent with depression. Propranolol is one of the most common
pharmacological agents to cause depression. Other side effects of
propranolol include dizziness, bronchospasm, nausea, vomiting, diarrhea,
and constipation. It may also precipitate asthma, congestive heart failure,
and hypoglycemia in susceptible patients. The patient should be switched to
another antihypertensive agent.
Just advising her to start a "gentle" walking routine to "lift her spirits" (choice
A) is inappropriate because she is having real depressive symptoms that are
most likely due to the propranolol.
It is inappropriate to explain to the patient that she is stressed about her trip,
and these symptoms will pass (choice C). She is obviously having real
symptoms that require serious attention, and in this case the depressive
symptoms are most likely due to propranolol.
It is inappropriate to give her a prescription for tranylcypromine (choice D)
because her depressive symptoms are most likely due to propranolol. Also,
tranylcypromine, which is an MAOI, is not the best first choice for an
antidepressive agent due to the necessary dietary restrictions and the risk of
orthostatic hypotension. If you were going to give her an antidepressant, an
SSRI or a tricyclic, such as nortriptyline, is better because of the relatively
low risk of orthostatic hypotension or anticholinergic effects.
Since propranolol is most likely causing her depressive symptoms, it is
incorrect to increase the dose of propranolol and prescribe phenelzine
(choice E). Also, phenelzine, which is an MAOI, is not the best first choice
for an antidepressive agent due to the necessary dietary restrictions and the
risk of orthostatic hypotension.
A 39-year-old woman comes to the office for a periodic health maintenance
examination. You have been treating her for anorexia nervosa for the past 15
years. She has required 4 hospitalizations during this time. She tells you that
she is "doing as well as can be expected" now and that she has a new
boyfriend who is very helpful. She is 157 cm (5 ft 2 in) tall and weighs 41 kg
46
(90 lb). She admits that she still does not have a menstrual period. Her
temperature is 36.7 C (98.0 F), blood pressure is 100/70 mm Hg, and her
pulse is 55/min. Physical examination shows lanugo, dry, scaly skin, yellow
discoloration of the palms, and pharyngeal abrasions. At this time the most
correct statement is:
A. Electrocardiography will show shortened QT intervals
B. Fluoxetine is not effective in reducing binge eating and purging
episodes
C. Her immune function is seriously affected and she is at risk for
severe infections
D. Laboratory studies will show hyperkalemia
E. She is at an increased risk for developing osteoporosis
Explanation:
The correct answer is E. This patient has anorexia nervosa with some signs
of purging (pharyngeal abrasion) and she is at an increased risk for
developing osteoporosis. The exact cause for decreased bone density in
unknown, but some believe that it is due to estrogen deficiency or cortisol
excess. It is interesting to note that giving estrogen replacement therapy to
these patients has not been shown to restore bone density.
Individuals with anorexia are at an increased risk for developing ventricular
arrhythmias and sudden death. An EKG often shows prolonged QT
intervals, not shortened QT intervals (choice A).
It is incorrect to say that fluoxetine is not effective in reducing binge eating
and purging episodes (choice B) because it has been shown to be effective.
While anorexia nervosa is sometimes associated with leukopenia and low
levels of IgG and IgM, severe immune dysfunction and severe infections are
very rare. Therefore, it is incorrect to say that her immune function is
seriously affected and that she is at risk for severe infections (choice C).
Hypokalemia, not hyperkalemia (choice D), is often found in patients with
anorexia nervosa. This is thought to be secondary to vomiting and other
behaviors common in anorexics.
A 35-year-old homeless man is admitted to the hospital because of psychosis
and agitation. He received intramuscular haloperidol while in the emergency
department 16 hours earlier. He is now obtunded, with a temperature of 39.1
C (102.4 F) and has muscular rigidity. He has had wide fluctuations in blood
pressure over the past 12 hours. Laboratory studies show a creatine
phosphokinase in excess of 20,000 U/L and a leukocyte count of
12,800/mm3. Supportive measures such as intravenous fluids and airway
support are given and the patient is noted to be hemodynamically stable. The
pharmacologic agent that would be contraindicated at this time is
47
A. acetaminophen
B. benztropine
C. bromocriptine
D. dantrolene
E. lorazepam
Explanation:
The correct answer is B. The patient has a diagnosis of neuroleptic
malignant syndrome due to rapid administration of high doses of
intramuscular haloperidol. The patient's relatively young age and male sex
are predispositions for the development of neuroleptic malignant syndrome,
though the biggest risk for any patient is the rapid administration of high
potency neuroleptics. Because of the patient's fever and autonomic
instability, benztropine, an anticholinergic medication with a tendency to
promote heat retention, would be contraindicated in a patient with
neuroleptic malignant syndrome.
Because of the high fevers associated with neuroleptic malignant syndrome,
an antipyretic such as acetaminophen (choice A) that is usually given
parenterally may be necessary.
Bromocriptine (choice C) is a dopamine agonist that is frequently used in the
treatment of neuroleptic malignant syndrome. One of the etiologic theories of
neuroleptic malignant syndrome is a dysregulation of dopamine receptors in
the hypothalamus resulting in impairment in body temperature control from
overblockade of hypothalamic dopamine receptors.
Dantrolene (choice D) is a muscle relaxant frequently used in the treatment
of neuroleptic malignant syndrome and may have efficacy in preventing
renal failure secondary to myoglobinuria.
Lorazepam (choice E) can be used in the treatment of neuroleptic malignant
syndrome both for its muscle relaxant and sedative qualities.
A 4-year-old girl is brought to the emergency department by her parents
because of pain and swelling in the girl's right forearm. The father says that
the girl was playing on a swing when she fell off and landed on her arm.
Physical examination reveals edema and erythema over the distal aspect the
right forearm. The girl does not want her arm to be touched and is crying
quietly. A radiograph of the right forearm demonstrates a small corner
fracture in the distal metaphysis of the radius. You are concerned about this
child because the head nurse tells you privately that this is the child's third
emergency department visit in the last 6 months. You decide to discuss your
concern for the child and the need for further evaluation with the parents. As
you are talking to them, the father becomes very angry and says he would
like to take his daughter home immediately. The most appropriate step for
48
you to take now is
A. admit the child to the pediatrics service for evaluation and
protection
B. keep the child in the emergency department and call the police to
report child abuse
C. keep the child in the emergency department until another relative or
family friend is found that can care for her
D. send the child home with her parents and call the police to report
child abuse
E. send the child to a foster care facility for protection
Explanation:
The correct answer is A. In all suspected cases of child abuse and neglect,
the physician has the authority to admit a child to the hospital for further
evaluation and protection against the wishes of the child's parents. In fact,
since the passage of the Child Abuse and Prevention Act, most states have
passed legislation to make the reporting of all cases of suspected child
abuse and neglect, by physicians and most healthcare workers, mandatory.
Signs of child abuse include multiple injuries that require visits to the
emergency department, fractures of varying ages, and “corner” fractures of
the metaphysis of long bones which are usually sustained by a twisting type
trauma. Since there is enough suspicion in this case for physical abuse, you
not only have the authority, but are required to protect this child and to report
the situation to the child protective service agency.
The most appropriate action is to admit the child for further evaluation and
protection. Keeping the child in the emergency department and calling the
police (choice B) does not address the problem of what to do with the child.
The appropriate authority to notify in suspected cases of child abuse and
neglect, is the child protective service agency.
Keeping the child in the emergency department until another relative can be
found (choice C) is not appropriate. The goal is to provide protection for the
child as well as to further evaluate her for signs of abuse. This cannot be
accomplished if the child is released with another adult. The most
appropriate action is to admit the child to the hospital for evaluation and
protection.
Sending the child home with her parents and calling the police (choice D) is
inappropriate in this case. There are enough signs of possible child abuse in
this case that the child should get further evaluation as well as protection
from further potential abuse. It is a physician's duty to protect the child by
admitting her to the hospital and calling the child protective service agency.
Sending the child to a foster care facility (choice E) at this time is premature.
Child abuse is suspected and the child should be admitted to the hospital for
further evaluation as well as protection from potential abuse. If child abuse is
established and if the home setting carries a high risk to the child's health,
temporary removal from the home is considered. Removal requires a court
49
petition, presented by the legal counsel of the appropriate welfare
department. The procedure varies from state to state, but usually entails a
family court testimony by a physician.
A 57-year-old man with schizoaffective disorder, whose symptoms were in
remission until 2 weeks ago, is brought to the emergency department by his
girlfriend. He will not talk with you but the girlfriend tells you that he has a
history of "overdoses", and she is afraid he has taken a lot of "his pills". He
has been complaining of voices telling him he "should be dead". He has not
left the house in a month and has spent several hours a day looking out the
window for the "king and savior" to "come take him". His medications include
haloperidol, valproic acid, and a small dose of amitriptyline for chronic pain
related to nerve damage in his leg, which occurred in a motor vehicle
accident 10 years ago. He has no other medical problems. A chart review
reveals that he has no allergies and was diagnosed with schizoaffective
disorder 30 years ago. His temperature is 37.0 C (98.6 F), blood pressure is
110/70 mm Hg, pulse is 70/min, and respirations are 26/min. He is a
depressed appearing man with very poor eye contact. He smells of alcohol.
He does not acknowledge you, but will answer some questions for his
girlfriend. He does admit to feeling that "life is not worth living" and feeling
"more religious than usual". He seems slightly drowsy and knows the date.
His physical examination is normal. The most appropriate next step in
management is to order
A. acetaminophen and salicylate levels
B. blood alcohol level
C. electrocardiography
D. urine toxicology for street drugs
E. valproic acid level
Explanation:
The correct answer is C. This patient may have taken several cardiotoxic
medications. Haloperidol and other antipsychotics increase the QT interval,
as do the tricyclics. In an overdose for either medication, he is at risk for
sudden death from torsades.
Acetaminophen and salicylate levels (choice A) and urine toxicology (choice
D) are helpful in patients suspected of intentional overdose, but as above,
risk for sudden death is the immediate concern.
A blood alcohol level (choice B) and a valproic acid level (choice E) are
reasonable choices for a lethargic patient, but this patient is awake and alert
and is not showing signs of CNS depression.
A 37-year-old man has been coming to you for a month for management of
50
depression that has not improved despite "years of wasted psychiatrist visits."
You prescribed a full dose of fluoxetine 4 weeks earlier after you ruled out a
medical etiology for his condition. He returns today to tell you that he has not
noticed any improvement in his mood, energy level, or ability to concentrate.
He promises you that he is taking the medication exactly as prescribed and
that he is not drinking any alcohol. Physical examination is unremarkable. He
denies any suicidal ideation and does not have any psychotic symptoms. He
really wants you to prescribe a different medication. At this time you should
A. add buspirone to his current treatment regimen
B. advise him to go back to his psychiatrist
C. explain that fluoxetine requires at least 5 weeks to reach a steady
state and that treatment response should be reevaluated in 2-4 weeks
D. recommend that he taper the medication over a week and initiate a
trial with a different agent
E. tell him that the medication is probably not working because he has
a negative attitude
Explanation:
The correct answer is C. Fluoxetine is a serotonin reuptake inhibitor that
requires 5 weeks to reach a steady state in the body and takes
approximately 6-8 weeks to show an adequate response. This should be
explained to patients before and during treatment to help them understand
the importance of staying on the medication even though they do not feel
any effects.
Adding buspirone to his current treatment regimen (choice A) may be the
appropriate management in the future but at this time, you should not make
any additions until the full response has been achieved (at least 6-8 weeks).
While a psychiatrist may be helpful, it does not seem like he was very
successful with his old psychiatrist. Advising him to go back to his
psychiatrist (choice B) is not the best idea. You should first try to explain that
he should give the medication a chance to work and if there is still no
improvement, a referral to a mental health specialist should be made.
Recommending that he taper the medication over a week and initiate a trial
with a different agent (choice D) is appropriate after unacceptable side
effects develop or after the 6 weeks, if the target dose of the drug results are
inadequate. It is too early to say that this medication is not working.
While a positive attitude is usually helpful, it is incorrect to tell him that the
medication is probably not working because he has a negative attitude
(choice E). "Blaming" the patient is inappropriate and it is not a good way to
maintain a good physician-patient relationship.
A 68-year-old man is admitted to the hospital for weight loss, anemia, and
worsening of his stomach pain that he thinks is due to an ulcer. However, the
51
results of tests that are performed confirm the diagnosis of an inoperable
cancer. You are told that when one of your residents tried to break the news
to him, he became mad, frustrated, and began to yell. He accused him of
being ignorant and threatened to sue him. You go to see the patient and he
says, "You think I am crazy and don't see what you are doing? They called
you so that I don't sue your young doctor for not knowing what he is doing.
You just want to help your buddies here, but you really can't help me". The
most appropriate management is to
A. clarify that the other physician is not a "buddy" and offer to help him
explore his decision about a lawsuit
B. empathize with his feelings of anger, grief, and fear indicating that
anger is a defense against intolerable emotions
C. explain that suing physicians is not always successful, and instead
suggest that the patient should take care of his affairs given his prognosis
D. offer to refer the patient to another team and physician in order to
help him find someone he can trust
E. sympathize with how badly he is treated and support the lawsuit
Explanation:
The correct answer is B. The appropriate intervention is to validate the
patient's feelings of anger and fear in order to help him deal with his
powerlessness in this situation. One must remain neutral, and yet
recognizes the difficulty of the patient's position.
Clarifying that physicians are not buddies (choice A) shows anger and
frustration towards the other physician. It is confrontational, and even though
exploring a lawsuit decision offers some comfort, this intervention with a
frustrated patient would only increase his anger.
Explaining that a lawsuit won't succeed and making sure that the patient
understands the fatality of his diagnosis (choice C) indicates the doctor's
frustration and the need to punish the "nasty patient". This intervention is
inappropriate and would likely increase the patient's frustration even more.
Offering to refer the patient to another team (choice D) is sending a
message of avoidance and confirming wrongdoing. It is not an intervention
that helps the patient deal with his feelings, but rather confirms his
suspicion.
Sympathizing (choice E) with the patient means recognizing the patient's
feelings as justified and taking his side. It is not helping, but rather
contributing to the patient's anger.
A 50-year-old woman comes to the office because of a "discrete right-sided
breast mass." She says that she discovered it in the shower 9 months ago,
and has been to 6 different physicians for evaluation. None of the other
physicians were able to palpate this " mass" and so they told her that it must
be "in her head." This is the first time that you are seeing this patient, so you
52
ask her to review her complete history. She is a married homemaker and is
the mother of 2 children who are both at out-of-town colleges. Her husband is
a traveling salesman. She has not had any medical problems in the past, and
her family history is unremarkable. Menarche was at age 14 and she is still
menstruating. She exercises 3 times a week, eats a low-fat diet, and drinks 13 glasses of wine a week. She has had normal Pap smears for the past 30
years and has never had a mammogram. She tearfully tells you that her best
friend died of breast cancer 2 years ago. She denies any episodes of
"sadness", insomnia, or feelings of hopelessness, helplessness, or guilt.
Physical examination is completely normal. You cannot palpate any breast
masses, even after she guides you to the "exact location" on the right breast.
You send her for a mammogram and a breast sonogram, and tell her that you
will let her know the results as soon as they return. The results, which return
in 2 days, show normal breast tissue and no abnormalities. You have your
nurse call the patient and tell her to schedule an appointment at her
convenience to discuss the results. Today, at the return visit, you explain the
results of the tests and she still seems to feel that there is "something there."
You try to be as patient as possible and tell her that there is most likely no
mass present. She looks at the ground and says that it "will show up" if you
order more studies. After you discuss the situation further, the most
appropriate next step is to
A. advise her to voluntarily commit herself to the hospital for
psychiatric evaluation
B. encourage her to get a "second opinion" from another general
physician
C. order a CT scan of the chest
D. order a fine needle aspiration of the right breast
E. schedule regular weekly 15 minute appointments and recommend
that she also see a psychiatrist
Explanation:
The correct answer is E. This patient most likely has hypochondriasis, which
is a disorder where the patient has a firm conviction of having a serious
illness, despite repeated evidence to the contrary. Even when presented
with definitive evidence, these patients remain convinced that they are ill.
Patience, compassion, regularly scheduled appointments, and collaboration
with a psychiatrist may be effective in treating these patients.
Since this patient is not depressed, suicidal, or homicidal, hospitalization
(choice A) is not indicated. She should probably go to talk to a psychiatrist,
and make regularly scheduled appointments to see you.
A "second opinion" (choice B) is not necessary in this case of
hypochondriasis because she has already seen 6 other physicians, and
does not seem to have a breast mass. She needs regularly scheduled
appointments, and she should see a psychiatrist.
Because the clinical breast examination, mammogram, ultrasound, and CT
53
scan were unremarkable, a CT scan (choice C) and a fine needle aspiration
(choice D) are not necessary. This patient most likely has hypochondriasis
and usually a thorough medical work-up is enough, but unnecessary
procedures should be discouraged. Also, a FNA is used when there is a
discrete mass, and there is nothing in this case.
A 32-year-old pregnant woman comes to the office because of "terrible
headaches." When asked to describe the headaches, she states that there is
just a "general, constant tenseness." She is unable to identify any specific
triggers. She has been coming to you for periodic health maintenance
examinations for the past few years, but has been going to an obstetrician
that her mother-in-law recommended for routine prenatal care. Over the
years, you have noticed that she has become more and more withdrawn, and
you have tried to gently approach the issue several times but she always
changes the subject. She has been married to a prominent lawyer, whom you
have never met, for the past 8 years. A neurologic examination is normal.
The medical gown falls open during the examination and you notice multiple
purple and yellowish-green ecchymoses on her breasts. When asked to tell
you about these findings, she looks down to the floor and quietly says that
she is "clumsy" and is "always banging into something." As she raises her
head, you notice that her cheeks are wet and that she is sniffling. The most
appropriate remark at this time is
A. "How long has your husband been abusing you?"
B. "I believe that your husband has been abusive for a while. Why
would you stay with him?"
C. "Those bruises are caused by a clotting abnormality that is
common during pregnancy"
D. "Why haven't you told me that your husband is abusive during our
previous appointments?"
E. "You need to leave your husband as soon as possible."
F. "You seem really upset about the circumstances under which you
got those bruises."
Explanation:
The correct answer is F. This patient is most likely a victim of spousal abuse
and it is important to call attention to the inconsistencies between her
response and her body language by saying, "You seem really upset about
the circumstances under which you got those bruises." It is usually good to
start out the conversation with an open-ended question, like "tell me about
these bruises." When she got visibly upset and made up an unlikely excuse,
it is appropriate to confront her (to point out that there is a discrepancy with
her statement and behavior). You can tell that she is upset because she is
looking down at the floor, speaking quietly, and obviously crying (wet cheeks
and sniffling). Spousal abuse is very common and women typically seek
medical attention for headaches, abdominal pain, pelvic pain, or depression.
It is rare that they come in complaining of spousal abuse. The physician
54
must be able to recognize the signs and symptoms of abuse.
It is not correct to ask her, "How long has your husband been abusing you?"
(choice A), "I believe that your husband has been abusive for a while. Why
would you stay with him?" (choice B) or "You need to leave your husband as
soon as possible" (choice E) because she has not yet told you that she has
been abused. To maintain a good doctor-patient relationship it is important
to allow the patient to feel comfortable and share information with you,
without feeling like you are jumping to conclusions. It is best to try to get her
to talk to you, rather than you automatically "blurting out" your opinions and
putting her on the defensive. A direct question, such as "does your husband
ever hit you?" may be necessary, but it is best to start out by allowing the
patient to describe the situation and speak freely.
From the physical examination and her body language, it seems more likely
that she is a victim of spousal abuse than a coagulopathy. Therefore it is
incorrect to tell her that "Those bruises are caused by a clotting abnormality
that is common during pregnancy" (choice C). Also, it is not normal to have
multiple bruises in various stages of healing on the breasts during
pregnancy.
It is inappropriate to ask her, "Why haven't you told me that your husband is
abusive during our previous appointments?" (choice D), because this
automatically puts her on the defensive. You should never "accuse" a patient
of not acting in the way which you think is best. She is obviously very upset
about the situation, and you will only make her feel worse by this statement.
Patients should not be criticized about their decisions. This will only alienate
them and destroy the doctor-patient relationship that is built on trust and
confidence.
You are notified that one of your patients, a 32-year-old woman, delivered a
healthy baby girl 6 hours earlier. You happen to be in the hospital discharging
another patient, so you go to the labor and delivery floor to see her. You have
been treating her for dysthymia and for a couple of episodes of major
depressive disorder over the years. She developed postpartum "blues" after
her last child was born that resolved spontaneously after 5 days. You hear a
woman screaming as you get off the elevator and head towards her room. As
you get closer, you recognize the voice and realize that it is your patient. She
is running around her room, tearing off the hospital gown, and yelling that,
"they are coming to get her." When she sees that you are standing in the
room, she begins to throw flower vases, the telephone, and the bedding at
you. The nurses appear and tell you that she has been very disorganized and
has had bizarre, grandiose delusions. She then goes back to the nurse's
station. You notice the newborn in the corner of the room. The most
appropriate next step is to
A. ask the nurse to get haloperidol from the medication closet
B. call for an immediate psychiatry consultation
55
C. encourage her to breastfeed and bond with her newborn
D. remove the newborn from the room
E. try to talk to her and calm her down
Explanation:
The correct answer is D. This woman is experiencing postpartum psychosis
and may harm herself and/or her newborn. You must immediately remove
the newborn from the room before the mother has a chance to harm her.
Postpartum psychosis is a relatively uncommon disorder that may affect
women with bipolar disorder, depression, and schizophrenia. It may also
occur in women with no previous psychiatric history. The treatment includes
the protection of the newborn, the administration of an antipsychotic
medication, and observation. Both suicide and infanticide may occur with
postpartum psychosis.
Haloperidol (choice A) is part of the acute treatment of postpartum
psychosis. However the action that is most likely to have the most
immediate effect is to remove the newborn from the room as fast as possible
and then to administer medication to the mother. The antipsychotic agent
will not have an immediate effect so it is first mandatory to protect the baby
from the mother's rage.
An immediate psychiatry consultation (choice B) may be necessary, but it is
important to first protect the newborn by removing her from the mother's
room.
At this time it is completely inappropriate to encourage her to breastfeed and
bond with her newborn (choice C). This woman most likely has postpartum
psychosis and is out of control. You must remove the newborn from the
room and then administer antipsychotic agents. She then needs close
observation. Encouraging bonding at this time may only worsen her
condition and endanger the life of the baby.
After you remove the baby from the room and administer an antipsychotic
agent, you can try to talk to her and calm her down (choice E). It will most
likely be useless to do this before the medication, and you first need to
remove the infant from this violent situation.
A 37-year-old obese man comes to the office because he has been "feeling
really bad lately." He says that for the past three months he has been having
trouble sleeping and has not been "in the mood" to go out. He has even
stopped going to basketball games with friends, which was his favorite hobby.
He has missed many days of work and finds it very difficult to concentrate. He
states that he feels "pretty helpless." All of his friends from college are
married with kids, and he says that he "can't even get a date," so he basically
gave up on having a family. He just feels "worthless". The most vital question
to ask at this time is
56
A. "Are you currently questioning your sexual orientation?"
B. "Do you ever feel like 'life really is not worth it and that you should
end it all'?"
C. "Do you think that your life would be much better if you were
dating?"
D. "Have any of your friends ever tried to set you up on a blind date?"
E. "Why haven't you tried to lose weight?"
Explanation:
The correct answer is B. This patient has symptoms suggestive of major
depressive disorder, and assessment of suicide risk is paramount in
interviewing him. Depressed patients have a significantly higher risk of
attempting suicide compared with the general population. Therefore,
determining whether a depressed patient is at high risk is essential early in
their management.
"Are you currently questioning your sexual orientation?" (choice A) is
incorrect because it is most important to ask about suicidal ideation at this
time, and it does not seem like this question applies to this case. It is
important for physicians to remain open about issues concerning sexuality,
and when one questions a patient about sexual orientation, it is appropriate
to remain nonjudgmental and ask, "Are you sexually active with women,
men, or both?" This, however, is not the most vital question in this case.
"Do you think that your life would be much better if you were dating?"
(choice C) and "Have any of your friends ever tried to set you up on a blind
date?" (choice D) are not vital questions at this time and will not immediately
affect your management. This patient has symptoms suggestive of major
depressive disorder and therefore must be asked about suicidal ideation.
"Why haven't you tried to lose weight?" (choice E) is an inappropriate
question because it sounds like you are accusing him of not trying to lose
weight. Also, his weight is not the most important issue at this time. While
obesity is associated with many serious medical conditions, his depression
may lead him to commit suicide soon, and it is therefore vital to assess his
risk.
A 54-year-old woman comes to your office. She has been your patient for
several years, and you usually see her for her annual physical. You have long
suspected depression because of her vague somatic complaints, but she has
previously denied being in a depressed mood or anhedonia. At this visit she
admits to feeling depressed "more often than not" and she is no longer
interested in her gardening club or playing with her grandchildren. She
tearfully reports increasing difficulty in falling asleep and exhaustion during
the day for the last several weeks. She has a decreased appetite and has lost
25 pounds in the last 3 months. She denies suicidality, homicidality, or
psychotic symptoms. She denies drug and alcohol use. She is a depressed-
57
appearing woman with very poor eye contact and moderate psychomotor
retardation. She scores a 29/30 on the Mental Status Exam. Her physical
examination is normal. You review the chart, and recall this patient has no
allergies and her only medication is estrogen replacement status post a total
hysterectomy 10 years ago. You order a TSH and decide to start an
antidepressant. You prescribe a selective serotonin reuptake inhibitor. You
should advise her that
A. she should call you if you she has any problems, but most people
have no difficulty with this medicine
B. early on, this medicine can cause headaches and nausea, while
later on, some people have a decreased desire for sex or an increase in
the time to orgasm
C. early on, this medicine can cause an increase in thirst and
urination, while later on, some people have a decreased desire for sex or
an increase in time to orgasm
D. early on, this medicine can cause weight gain and hair loss, while
later on, some people have a decreased desire for sex or an increase in
the time to orgasm
E. early on, this medicine can cause muscle rigidity and tremors, while
later on, some people have a decreased desire for sex or an increase in
the time to orgasm
Explanation:
The correct answer is B. Headaches and GI distress are common side
effects that usually resolve in the first week of treatment. If the patient
cannot tolerate a lower dose, change the medicine. Sexual side effects
usually begin a few weeks into treatment.
She should call you if you she has any problems, but most people have no
difficulty with this medicine (choice A) is incorrect because sexual side
effects should be described specifically, and discussed during the follow up
visits, although many people tolerate the SSRIs without any problems.
Sexual side effects are the most common reason SSRIs are stopped.
Obviously she should be told to call if there are any problems, but first you
should advise her about the common side effects.
Early on, this medicine can cause an increase in thirst and urination, while
later on, some people have a decrease in desire for sex or an increase in
time to orgasm (choice C) is incorrect. Lithium can cause diabetes insipidus
and patients will report an increase in thirst and urination.
Early on, this medicine can cause weight gain and hair loss, while later on
some people have a decreased desire for sex or an increase in the time to
orgasm (choice D) and early on this medicine can cause muscle rigidity and
a tremor (choice E) is incorrect. Antipsychotic medicines can cause
parkinsonian symptoms. Patients will have bradykinesia, rigidity, postural
instability, and tremor.
58
A 35-year-old woman comes to the office asking you to drug test her 12-year-old son.
She states that her son is normally a very kind and interactive child, however, for the last
3 months, he has become increasingly withdrawn. He is in his room most of the time
except for when he is at school. His grades have dropped from an A- average to C- this
past semester. He refuses to see any of his friends and does not even eat much during
dinner anymore. She has confronted her son multiple times about his situation, but he
continually denies everything, including drug or alcohol use. The mother is visibly upset
at this situation and is tearful about it. She tells you that you are her son's "only help."
The most appropriate response to the mother's request is:
A. "Just bring your son in and we can then get a urine test for alcohol and
substance use."
B. "I am sorry but I can't legally test your son for any drugs without him consenting
to it first."
C. "It is possible that your son may be suffering from depression and I think that
you should bring him in for me to talk to him."
D. "These are classic symptoms for schizophrenia. The best thing to do would be
to start him on some anti-psychotic medications."
E. "Your son is most probably abusing drugs and alcohol, so you should send him
to a rehabilitation facility immediately."
Explanation:
The correct answer is C. The mother is right to be concerned over her son's change in
behavior. Certainly drugs and alcohol abuse can cause such dramatic behavioral
changes. However, more likely would be a mental illness such as depression. The
DSM–IV criteria for major depressive episode includes that at least one of the symptoms
is either (1) depressed mood or (2) loss of interest in pleasure. The patient would also
need to have 5 (or more) of the symptoms such as depressed mood, weight loss,
insomnia, fatigue, suicidal tendencies, etc. Additionally, the symptoms should cause
clinically significant distress or social impairment. It would also be important to screen
the son for suicidality when the mother brings him in.
"Just bring your son in and we can then get a urine test for alcohol and substance use."
(choice A) would be appropriate if the son consented to the drug test. However, covert
use of the son's urine would only cause him to distrust not only his mother but also the
physician, making any future interventions more difficult.
"I'm sorry but I can't legally test your son for any drugs without him consenting to it first."
(choice B) is not true because the child is a minor and the parent is consenting to a drug
test. However, it wouldn't be going to the root of this child's problem since he most likely
suffers from a depressive disorder.
"These are classic symptoms for schizophrenia. The best thing to do would be to start
him on some anti-psychotic medications." (choice D) would be a premature statement
since the diagnosis of schizophrenia requires both positive and negative symptoms
occurring for 6 months. However, further questioning of the child may reveal that he is
having a beginning prodrome for schizophrenia and starting him on anti-psychotics may
be appropriate in the future.
"Your son is most probably abusing drugs and alcohol, so you should send him to a
rehabilitation facility immediately." (choice E) is inappropriate given the high likelihood
59
that the son is suffering from depression. It is important to make sure that his depression
is not due to the direct effects of a substance (e.g., a drug of abuse, a medication).
However, evidence of substance abuse should first be obtained through discussion with
the son or consented toxicology screens.
60